Sunteți pe pagina 1din 44

DC CIRCUITS

19

Responses to Questions
1.

Even though the birds feet are at high potential with respect to the ground, there is very little potential
difference between them, because they are close together on the wire. The resistance of the bird is
much greater than the resistance of the wire between the birds feet. These two resistances are in
parallel, so very little current will pass through the bird as it perches on the wire. However, when you
put a metal ladder up against a power line, you provide a direct connection between the high potential
line and ground. The ladder will have a large potential difference between its top and bottom. A person
standing on the ladder will also have a large potential difference between his or her hands and feet.
Even if the persons resistance is large, the potential difference will be great enough to produce a
current through the persons body large enough to cause substantial damage or death.

2.

Series: The main disadvantage of Christmas tree lights connected in series is that when one bulb burns
out, a gap is created in the circuit and none of the bulbs remains lit. Finding the burned-out bulb
requires replacing each individual bulb one at a time until the string of bulbs comes back on. As an
advantage, the bulbs are slightly easier to wire in series. Only one wire needs to be attached from bulb
to bulb. Also, a blinker bulb can make the entire string flash on and off by cutting off the current.
Parallel: The main advantage of connecting the bulbs in parallel is that one burned-out bulb does not
affect the rest of the strand and is easy to identify and replace. As a disadvantage, wiring the bulbs in
parallel is slightly more difficult. Two wires must be attached from bulb to bulb.

3.

If 20 of the 6-V lamps were connected in series and then connected to the 120-V line, there would be a
voltage drop of 6 V for each of the lamps, and they would not burn out due to too much voltage. Being
in series, if one of the bulbs went out for any reason, then they would all turn off.

4.

If the lightbulbs are in series, then each will have the same current. The power dissipated by the bulb
as heat and light is given by P = I 2 R. Thus the bulb with the higher resistance ( R2 ) will be brighter.
If the bulbs are in parallel, then each will have the same voltage. The power dissipated by the bulb as
heat and light is given by P = V 2 /R. Thus the bulb with the lower resistance ( R1 ) will be brighter.

5.

The outlets are connected in parallel to each other, because you can use one outlet without using the
other. If they were in series, then both outlets would have to be used at the same time to have a
completed circuit. Also, both outlets supply the same voltage to whatever device is plugged in to the
outlet, which indicates that they are wired in parallel to the voltage source.

Copyright 2014 Pearson Education, Inc. All rights reserved. This material is protected under all copyright laws as they currently exist.
No portion of this material may be reproduced, in any form or by any means, without permission in writing from the publisher.

19-1

19-2

Chapter 19

6.

The power output from a resistor is given by P = V 2 /R. To maximize this value, the voltage needs to
be as large as possible and the resistance as small as possible. That can be accomplished by putting the
two batteries in series and then connecting the two resistors in parallel to each other, across the full
two-battery voltage.

7.

The battery has to supply less power when the two resistors are connected in series than it has to
supply when only one resistor is connected. P = V 2 /R, so if V is constant and R increases, then the
power decreases.

8.

We assume that the bulbs are in parallel with each other. The overall resistance decreases and more
current is drawn from the source. A bulb rated at 60 W and 120 V has a resistance of 240 . A bulb
rated at 100 W and 120 V has a resistance of 144 . When only the 60-W bulb is on, the total
resistance is 240 . When both bulbs are lit, the total resistance is the combination of the two
resistances in parallel, which is only 90 .

9.

The energy stored in a capacitor network can be calculated by PE = 12 CV 2 . Since the voltage for the
capacitor network is the same in this problem for both configurations, the configuration with the
highest equivalent capacitance will store the most energy. The parallel combination has the highest
equivalent capacitance, so it stores the most energy. Another way to consider this is that the total
stored energy is the sum of the quantity PE = 12 CV 2 for each capacitor. Each capacitor has the same
capacitance, but in the parallel circuit, each capacitor has a larger voltage than in the series circuit.
Thus the parallel circuit stores more energy.

10.

No, the sign of the batterys emf does not depend on the direction of the current through the battery.
The sign of the batterys emf depends on the direction you go through the battery in applying the loop
rule. If you go from negative pole to positive pole, then the emf is added. If you go from positive pole
to negative pole, then the emf is subtracted.
But the terminal voltage does depend on the direction of the current through the battery. If current is
flowing through the battery in the normal orientation (leaving the positive terminal, flowing through
the circuit, and arriving at the negative terminal), then there is a voltage drop across the internal
resistance, and the terminal voltage is less than the emf. If the current flows in the opposite sense (as in
charging the battery), then there is a voltage rise across the terminal resistance, and the terminal
voltage is higher than the emf.

11.

(a)
(b)

With the batteries in series, a greater voltage is delivered to the lamp, and the lamp will burn
brighter.
With the batteries in parallel, the voltage across the lamp is the same as for either battery alone.
Each battery supplies only half of the current going through the lamp, so the batteries will last
longer (and the bulb stay lit longer) as compared to just having one battery.

12.

When batteries are connected in series, their emfs add together, producing a larger potential. For
instance, if there are two 1.5-V batteries in series in a flashlight, then the potential across the bulb will
be 3.0 V, and there will be more current in the bulb. The batteries do not need to be identical in this
case. When batteries are connected in parallel, the currents they can generate add together, producing a
larger current over a longer time period. Batteries in this case need to be nearly identical, or the battery
with the larger emf will end up charging the battery with the smaller emf.

13.

Yes. When a battery is being charged, current is forced through it backward and Vterminal = + Ir ,
so Vterminal > .

Copyright 2014 Pearson Education, Inc. All rights reserved. This material is protected under all copyright laws as they currently exist.
No portion of this material may be reproduced, in any form or by any means, without permission in writing from the publisher.

DC Circuits

14.

19-3

Refer to Fig. 192. Connect the battery to a known resistance R and measure the terminal voltage Vab .
Vab
. It is also true that Vab = Ir , so the
R
Vab
=R
.
Vab

The current in the circuit is given by Ohms law to be I =


internal resistance can be calculated by r =

Vab
I

15.

No. As current passes through the resistor in the RC circuit, energy is dissipated in the resistor.
Therefore, the total energy supplied by the battery during the charging is the combination of the energy
dissipated in the resistor and the energy stored in the capacitor.

16.

(a)
(f)

17.

The following is one way to accomplish the desired task.

stays the same; (b)


decreases;
(g)

increases; (c) decreases;


decreases; (h) increases;

(d) increases;
(e) increases;
(i) stays the same.

Light

In the current configuration, the light would be on. If either switch is moved, then the light will go out.
But if either switch were moved again, then the light would come back on.
18.

The soles of your shoes are made of a material which has a relatively high resistance, and there is
relatively high resistance flooring material between your shoes and the literal ground (the Earth). With
that high resistance, a malfunctioning appliance would not be able to cause a large current flow
through your body. The resistance of bare feet is much less than that of shoes, and the feet are in direct
contact with the ground, so the total resistance is much lower and a larger current would flow through
your body.

19.

In an analog ammeter, the internal resistor, or shunt resistor, has a small value and is in parallel with
the galvanometer, so that the overall resistance of the ammeter is very small. In an analog voltmeter,
the internal resistor has a large value and is in series with the galvanometer, and the overall resistance
of the voltmeter is very large.

20.

If you mistakenly use an ammeter where you intend to use a voltmeter, then you are inserting a short in
parallel with some resistance. That means that the resistance of the entire circuit has been lowered, and
almost all of the current will flow through the low-resistance ammeter. Ammeters usually have a fairly
small current limit, so the ammeter will very likely be damaged in such a scenario. Also, if the
ammeter is inserted across a voltage source, then the source will provide a large current, and again the
meter will almost certainly be damaged or at least disabled by burning out a fuse.

21.

An ammeter is placed in series with a given circuit element in order to measure the current through
that element. If the ammeter did not have very low (ideally, zero) resistance, then its presence in the
circuit would change the current it is attempting to measure by adding more resistance in series. An
ideal ammeter has zero resistance and thus does not change the current it is measuring.
A voltmeter is placed in parallel with a circuit element in order to measure the voltage difference
across that element. If the voltmeter does not have a very high resistance, then its presence in parallel
will lower the overall resistance and affect the circuit. An ideal voltmeter has infinite resistance so that
when placed in parallel with circuit elements it will not change the value of the voltage it is reading.

Copyright 2014 Pearson Education, Inc. All rights reserved. This material is protected under all copyright laws as they currently exist.
No portion of this material may be reproduced, in any form or by any means, without permission in writing from the publisher.

19-4

Chapter 19

22.

When the voltmeter is connected to the circuit, it reduces the resistance of that part of the circuit. That
will make the resistor + voltmeter combination a smaller fraction of the total resistance of the circuit
than the resistor was alone, which means that it will have a smaller fraction of the total voltage drop
across it.

23.

A voltmeter has a very high resistance. When it is connected to the battery, very little current will flow.
A small current results in a small voltage drop due to the internal resistance of the battery, and the emf
and terminal voltage (measured by the voltmeter) will be very close to the same value. However, when
the battery is connected to the lower-resistance flashlight bulb, the current will be higher and the
voltage drop due to the internal resistance of the battery will also be higher. As a battery is used, its
internal resistance increases. Therefore, the terminal voltage will be significantly lower than the emf:
Vterminal = Ir. A lower terminal voltage will result in a dimmer bulb and usually indicates a usedup battery.

Responses to MisConceptual Questions


1.

(a, c) Resistors are connected in series when there are no junctions between the resistors. With no
junctions between the resistors, any current flowing through one of the resistors must also flow
through the other resistor.

2.

(d)

It might be easy to think that all three resistors are in parallel because they are on three branches
of the circuit. However, following a path from the positive terminal of the battery, all of the
current from the battery passes through R2 before reaching the junction at the end of that
branch. The current then splits and part of the current passes through R1 while the remainder of
the current passes through R3 before meeting at the right junction.

3.

(c)

A common misconception is that the smaller resistor would have the larger current. However,
the two resistors are in series, so they must have the same current flowing through them. It does
not matter which resistor is first.

4.

(a)

It might seem that current must flow through each resistor and lightbulb. However, current will
only flow through the lightbulb if there is a potential difference across the bulb. If we consider
the bottom branch of the circuit to be at ground, then the left end of the lightbulb will be at a
potential of 10 V. The top branch will be at 20 V. Since the two resistors are identical, the
voltage drop across each will be half the voltage from the top to bottom, or 10 V each. This
makes the right side of the lightbulb also at 10 V. Since both sides of the lightbulb are at the
same potential, no current will flow through the lightbulb.

5.

(d)

Resistors R1 and R2 are in parallel, so each has half of the current from the battery. R3 and R4
are in series and add to produce twice as much resistance as R5 . Since they are in parallel
with R5 , one-third of the current from the battery goes through them, while two-thirds goes
through R5 . The greatest current therefore goes through R5 .

6.

(d)

Current takes the path of least resistance. Since bulb A is in parallel with the short circuit, all of
the current will pass through the short circuit, causing bulb A to go out.

Copyright 2014 Pearson Education, Inc. All rights reserved. This material is protected under all copyright laws as they currently exist.
No portion of this material may be reproduced, in any form or by any means, without permission in writing from the publisher.

DC Circuits

19-5

7.

(b)

A common misconception is that adding another parallel branch would have no effect on the
voltage across R4 . However, when the switch is closed, the additional parallel resistor makes the
effective resistance of the parallel resistors smaller, and therefore the resistance of the entire
circuit gets smaller. With a smaller effective resistance, a greater current flows through the
battery and through R1 , resulting in a greater voltage drop across R1. Since the voltage from the
battery is equal to the sum of the voltages across R1 and R4 , increasing the voltage across R1
results in a decrease in voltage across R4 .

8.

(a)

As explained in the answer to Question 7, when the switch is closed, it adds an additional
resistor in parallel to R3 and R4 , making the effective resistance of the entire circuit smaller.
With less effective resistance, a greater current flows through the circuit, increasing the potential
difference across R1.

9.

(b)

As the capacitor charges, the voltage drop across the capacitor increases, thereby diminishing the
voltage drop across the resistor. As the voltage drop across the resistor decreases, the current
decreases.

10.

(c)

Even though steady current cannot flow through a capacitor, charge can build up on the
capacitor, allowing current to initially flow in the circuit. As the charge builds on the capacitor,
the voltage drop across the capacitor increases, and the current decreases. The rate of charging is
determined by the time constant, which is the product of R and C.

11.

(b)

It might seem that a capacitor would discharge linearly in time, losing one-fourth of the charge
every second. This is incorrect, as the capacitor discharges exponentially. That is, every 2.0
seconds, half of the remaining charge on the capacitor will discharge. After 2.0 seconds, half of
the charge remains. After 4.0 seconds, half of the half, or one-fourth, of the charge remains.
After 6.0 seconds, one-eighth of the charge remains.

12.

(a)

To double the heart rate, the time of discharging must be shorter, so the discharge rate must be
faster. The resistance limits the rate at which the current can flow through the circuit. Decreasing
the resistance will increase the current flow, causing the capacitor to discharge faster.

13.

(c)

In a two-prong cord, one prong is at high voltage and the other is grounded. Electricity flows
through the appliance between these two wires. However, if there is a short between the highvoltage wire and the casing, then the casing can become charged and electrocute a person
touching the case. The third prong connects the external case to ground, so that the case cannot
become charged.

14.

(c)

Connecting capacitors in series effectively increases the plate separation distance, decreasing the
net capacitance. Connecting capacitors in parallel effectively increases the plate area, increasing
the net capacitance. Therefore, when capacitors are connected in series, the effective capacitance
will be less than the capacitance of the smallest capacitor, and when connected in parallel, the
effective capacitance will be greater than the capacitance of the largest capacitance.

15.

(b)

The ammeter is placed in series with the circuit and therefore should have a small resistance, so
there is minimal voltage drop across the ammeter. The voltmeter is placed in parallel with the
circuit. It should have a large resistance so that minimal current from the circuit passes through
the voltmeter instead of passing through the circuit.

Copyright 2014 Pearson Education, Inc. All rights reserved. This material is protected under all copyright laws as they currently exist.
No portion of this material may be reproduced, in any form or by any means, without permission in writing from the publisher.

19-6

Chapter 19

Solutions to Problems
1.

See Fig. 192 for a circuit diagram for this problem. Using the same analysis as in Example 191, the

. Use Eq. 191 to calculate the terminal voltage.


current in the circuit is I =
R+r
R
71.0
(R + r ) r

=
= (6.00 V)
= 5.92 V
(a) Vab = Ir =
r =
R
r
R
r
R
r
(71
0
+
+
+
.
+ 0.900)

(b)

2.

Vab =

R
710
= (6.00 V)
= 5.99 V
R+r
(710 + 0.900)

See the circuit diagram. The current in the circuit is I. The voltage Vab is given by Ohms law to be
Vab = IR. That same voltage is the terminal voltage of the series emf.

R
Vab = ( Ir ) + ( Ir ) + ( Ir ) + ( Ir ) = 4( Ir ) and Vab = IR
4( Ir ) = IR r =

14 IR
I

(1.50 V) 14 (0.45 A)(12.0 )


0.45 A

3.

See Fig. 192 for a circuit diagram for this problem. Use Eq. 191.
Vab 12.0 V 8.8 V
Vab = Ir r =
=
= 0.034
I
95 A
V
8.8 V
Vab = IR R = ab =
= 0.093
I
95 A

4.

The equivalent resistance is the sum of the two resistances:


Req = R1 + R2 . The current in the circuit is then the voltage
divided by the equivalent resistance: I =

=
. The
Req R1 + R2

= 0.333 0.33

R2

voltage across the 1800- resistor is given by Ohms law.


R2

1800
V2200 = IR2 =
R2 =
= (12 V)
= 8.8 V
R1 + R2
R1 + R2
650 + 1800
5.

R1

(a)

For the resistors in series, use Eq. 193, which says the resistances add linearly.
Req = 3(45 ) + 3(65 ) = 330

(b)

For the resistors in parallel, use Eq. 194, which says the resistances add reciprocally.
1
1
1
1
1
1
1
3
3
3(65 ) + 3(45 )
=
+
+
+
+
+
=
+
=
Req 45 45 45 65 65 65 45 65
(65 )(45 )
Req =

(65 )(45 )
= 8.9
3(65 ) + 3(45 )

Copyright 2014 Pearson Education, Inc. All rights reserved. This material is protected under all copyright laws as they currently exist.
No portion of this material may be reproduced, in any form or by any means, without permission in writing from the publisher.

DC Circuits

6.

(a)

19-7

The maximum resistance is made by combining the resistors in series.


Req = R1 + R2 + R3 = 580 + 790 + 1200 = 2.57 k

(b)

The minimum resistance is made by combining the resistors in parallel.


1
1
1
1
=
+
+
Req R1 R2 R3

1
1
1
Req = +
+

R
R
R
2
3
1

7.

1
1
1
1
1
= +
+
+
+

R1 R2 R3 R4 R5

= 260

5
=

100

= 20 = n(10 ) n =

20
= 2
10

R1 = 40

R2 = 10
a

(40) = 0.8%, and the voltage across R2 would be V2 = IR2 = (10) = 0.2%.
50
50

Connecting 18 of the resistors in series will enable you to


make a voltage divider with a 3.5-V output. To get the desired
output, measure the voltage across 7 consecutive series
resistors. In the diagram that would be the voltage between
points a and b.

Req = 18(1.0 ) I =
=
Req 18.0
Vab = (7.0 ) I = (7.0 )

10.

The ratio R1 /R2 must be 4/1. Since both resistors have the same
current, the voltage drop across R1 (4/5 of the battery voltage)
will be 4 times that across R2 (1/5 of the battery voltage). So if
R1 = 40 and R2 = 10 , then the current would be /50.
The voltage across R1 would be found from Ohms law to be
V1 = IR1 =

9.

1
1
1
=
+
+

580
790
1200

The equivalent resistance of five 100- resistors in parallel is found, and then that resistance is
divided by 10 to find the number of 10- resistors needed.
Req

8.

11.0

7.0
b

9.0 V
= (7.0 )
= 3.5 V
18.0
18.0

The resistors can all be connected in series.


Req = R + R + R = 3(1.70 k) = 5.10 k

The resistors can all be connected in parallel.


1
1 1 1
= + +
Req R R R

3
Req =
R

R 1.70 k
=
= 567
3
3

Two resistors in series can be placed in parallel with the third.


1
1
1
1 1
3
= +
= +
=
Req R R + R R 2 R 2 R

Req =

2 R 2(1.70 k)
=
= 1.13 k
3
3

Copyright 2014 Pearson Education, Inc. All rights reserved. This material is protected under all copyright laws as they currently exist.
No portion of this material may be reproduced, in any form or by any means, without permission in writing from the publisher.

19-8

Chapter 19

Two resistors in parallel can be placed in series with the third.


1 1
Req = R + +
R R

11.

= R+

R 3
= (1.70 k) = 2.55 k
2 2

The resistance of each bulb can be found from its power rating.
V2
R

P=

R=

V 2 (12.0 V) 2
=
= 36
4.0 W
P

Find the equivalent resistance of the two bulbs in parallel.


1
1 1 2
= + =
Req R R R

Req =

R 36
=
= 18
2
2

The terminal voltage is the voltage across this equivalent resistance. Use that to find the current drawn
from the battery.
Vab = IReq

I=

Vab Vab 2Vab


=
=
Req R /2
R

Finally, use the terminal voltage and the current to find the internal resistance, as in Eq. 191.
Vab = Ir

12.

(a)

r=

Vab Vab
Vab
12.0 V 11.8 V
=
=R
= (36 )
= 0.305 0.3
2
V
2Vab
2(11.8 V)
I
ab
R

Each bulb should get one-eighth of the total voltage, but let us prove that instead of assuming it.
Since the bulbs are identical, the net resistance is Req = 8 R. The current flowing through the
bulbs is then Vtot = IReq

I=

Vtot Vtot
=
. The voltage across one bulb is found from Ohms
Req 8 R

law.
V = IR =

(b)

I=

Vtot
8R

Vtot
V
120 V
R = tot =
= 15 V
8R
8
8

R=

Vtot
120 V
=
= 33.33 33
8I
8(0.45 A)

P = I 2 R = (0.45 A) 2 (33.33 ) = 6.75 W 6.8 W

13.

We model the resistance of the long leads as a single resistor r. Since the bulbs are in parallel, the total
current is the sum of the current in each bulb, so I = 8I R . The voltage drop across the long leads is
Vleads = Ir = 8I R r = 8(0.21 A)(1.4 ) = 2.352 V. Thus the voltage across each of the parallel resistors is
VR = Vtot Vleads = 120 V 2.352 V = 117.6 V. Since we have the current through each resistor, and
the voltage across each resistor, we calculate the resistance using Ohms law.
VR = I R R R =

VR 117.6 V
=
= 560
IR
0.21 A

The total power delivered is P = Vtot I , and the wasted power is I 2 r. The fraction wasted is the ratio
of those powers.

Copyright 2014 Pearson Education, Inc. All rights reserved. This material is protected under all copyright laws as they currently exist.
No portion of this material may be reproduced, in any form or by any means, without permission in writing from the publisher.

DC Circuits

fraction wasted =

19-9

I 2r
Ir
8(0.21 A)(1.4 )
=
=
= 0.0196 0.020
120 V
IVtot Vtot

So about 2% of the power is wasted.


14.

To fix this circuit, connect another resistor in parallel with the 480- resistor so that the equivalent
resistance is the desired 350 .
1
1
R2 =

Req R1

1
1
1
=
+
Req R1 R2

1
1
=

350 480

= 1292 1300

So solder a 1300- resistor in parallel with the 480- resistor.


15.

Vtot
. That could be proven using a
8
similar argument to that used in Problem 12. Use that voltage and the power dissipated by each bulb to
calculate the resistance of a bulb.

Each bulb will get one-eighth of the total voltage, so Vbulb =

Pbulb =

16.

(a)

2
Vbulb
R

R=

2
Vbulb
V2
(120 V) 2
= tot =
= 32
P
64 P 64(7.0 W)

The equivalent resistance is found by combining the 750- and 680- resistors in parallel and
then adding the 990- resistor in series with that parallel combination.
1
1
Req =
+

750 680

(b)

+ 990 = 357 + 990 = 1347 1350

The current delivered by the battery is I =

V
12.0 V
=
= 8.909 103 A. This is the current in
Req 1347

the 990- resistor. The voltage across that resistor can be found by Ohms law.
V990 = IR = (8.909 103 A)(990 ) = 8.820 V 8.8 V

Thus the voltage across the parallel combination must be 12.0 V 8.8 V = 3.2 V . This is the
voltage across both the 750- and 680- resistors, since parallel resistors have the same
voltage across them. Note that this voltage value could also be found as follows.
Vparallel = IRparallel = (8.909 103 A)(357 ) = 3.181 V 3.2 V

(c)

The current in the 990- resistor was already seen to be 8.91 mA. The other currents can be
found from Ohms law.
I 750 =

V750 3.18 V
=
4.2 mA ;
R750 470

I 680 =

V750 3.18 V
=
4.7 mA
R680 680

Note that these last two currents add to be the current in the 990- resistor.
17.

The resistance of each bulb can be found by using Eq. 186, P = V 2 /R. The two individual resistances
are combined in parallel. We label the bulbs by their wattage.
P = V 2 /R

1
P
= 2
R V

Copyright 2014 Pearson Education, Inc. All rights reserved. This material is protected under all copyright laws as they currently exist.
No portion of this material may be reproduced, in any form or by any means, without permission in writing from the publisher.

19-10

Chapter 19

Req

18.

(a)

1
1
=
+

R75 R25

75 W
25 W
=
+
(120 V) 2 (120 V) 2

= 144 140

The three resistors on the far right are in series, so their equivalent
resistance is 3R. That combination is in parallel with the next
resistor to the left, as shown in the dashed box in figure (b). The
equivalent resistance of the dashed box is found as follows.
1 1
Req1 = +

R 3R

= 34 R

This equivalent resistance of

3
4

R is in series with the next two

resistors, as shown in the dashed box in figure (c). The equivalent


R. This
resistance of that dashed box is Req2 = 2 R + 34 R = 11
4
11 R
4

is in parallel with the next resistor to the left, as shown in

figure (d). The equivalent resistance of that dashed box is found


as follows.
4
1
Req2 = +

R 11R

11 R
= 15

This is in series with the last two resistors, the ones connected
directly to A and B. The final equivalent resistance is then
11 R = 41 R = 41 (175 ) = 478.33 478
Req = 2 R + 15
15
15

(b)

The current flowing from the battery is found from Ohms law.
V
50.0 V
=
= 0.10453 A 0.105 A
Req 478.33

I total =

This is the current in the top and bottom resistors. There will be less current in the next resistor
because the current splits, with some current passing through the resistor in question, and the rest
of the current passing through the equivalent resistance of 11
R, as shown in figure (d). The
4
voltage across R and across

11 R
4

must be the same, since they are in parallel. Use this to find the

desired current.
VR = V11 R
4

I R R = I 11 R
4

( 114 R ) = (I total I R ) ( 114 R )

11 I
11
I R = 15
total = 15 (0.10453 A)I total = 0.0767 A

19.

The resistors have been numbered in the accompanying diagram to help in the
analysis. R1 and R2 are in series with an equivalent resistance of
R12 = R + R = 2 R. This combination is in parallel with R3 , with an equivalent
1

1 1
2
resistance of R123 = +
= 3 R. This combination is in series with R4 ,
R 2R
with an equivalent resistance of R1234 = 32 R + R = 53 R. This combination is in
1 3
parallel with R5 , with an equivalent resistance of R12345 = +

R 5R

= 85 R.

R2
R1
A

R3
R5

R4
C

R6

Copyright 2014 Pearson Education, Inc. All rights reserved. This material is protected under all copyright laws as they currently exist.
No portion of this material may be reproduced, in any form or by any means, without permission in writing from the publisher.

DC Circuits

19-11

Finally, this combination is in series with R6 , and we calculate the final equivalent resistance.
Req = 85 R + R =

13 R
8

A more detailed solution with more circuit diagrams is given in Problem 20.
20.

We reduce the circuit to a single loop by combining


series and parallel combinations. We label a combined
resistance with the subscripts of the resistors used in
the combination. See the successive diagrams. The
initial diagram has been rotated by 90 degrees.
R1 and R2 are in series.

R2
R1
A

R3

R4
C

R5

R12
R3
A

R5

R6

R4
C

R6

R12 = R1 + R2 = R + R = 2 R

R123

1
1
=
+

R12 R3

R12 and R3 are in parallel.

1 1
=
+
2R R

B
= 23 R

R123
A

R123 and R4 are in series.


R1234 = R123 + R4 = 23 R + R = 53 R
R1234 and R5 are in parallel.

1
1
+
R12345 =

R
R
5
1234

1
1
=5 +
R R
3

Req = R12345 + R6 = R + R =

13
8

R5

R6

= 85 R

R12345

R6

R6

R12345 and R6 are in series, producing the equivalent


resistance.
5
8

R5

R1234

R4

Req

Now work backward from the simplified circuit. Resistors in series have the same current as their
equivalent resistance, and resistors in parallel have the same voltage as their equivalent resistance. To
avoid rounding errors, we do not use numeric values until the end of the problem.
I eq =

8
=
=
= I 6 = I12345
Req 13
13
R
R
8

8 5
5
V5 = V1234 = V12345 = I12345 R12345 =
( R) = 13 ;
13R 8
I1234 =

I3 =

V1234
=
R1234

5
13
5R
3

V3
2
=
= I3 ;
R3 13R

3
= I 4 = I123 ;
13R
I12 =

I5 =

5
V5 13
5
=
=
= I5
R5
R
13R

3 2
2
V123 = I123 R123 =
( R) = 13 = V12 = V3
13R 3

V12 13

=
=
= I1 = I 2
R12
2R
13R

Copyright 2014 Pearson Education, Inc. All rights reserved. This material is protected under all copyright laws as they currently exist.
No portion of this material may be reproduced, in any form or by any means, without permission in writing from the publisher.

19-12

Chapter 19

Now substitute in numerical values.


I1 = I 2 =
I5 =

21.

12.0 V
2
3
=
= 0.284 mA ; I3 =
= 0.568 mA ; I 4 =
= 0.852 mA ;
13R 13(3250 )
13R
13R

5
8
2 = 1.85 V
= 1.42 mA ; I 6 =
= 2.27 mA ; VAB = V3 = 13
13R
13R

It is given that the power used when the resistors are in series is one-fourth the power used when the
resistors are in parallel. The voltage is the same in both cases. Use Eq. 186b, along with the
definitions of series and parallel equivalent resistance.
Pseries = 14 Pparallel

V2
=
Rseries

( R1 + R2 ) 2 = 4 R1 R2

1
4

V2
Rparallel

Rseries = 4 Rparallel

( R1 + R2 ) = 4

R12 + 2 R1 R2 + R22 4 R1 R2 = 0 = ( R1 R2 ) 2

R1 R2
( R1 + R2 )

R1 = R2

Thus the two resistors must be the same, so the other resistor is 4.8 k .
22.

We label identical resistors from left to right as Rleft , Rmiddle , and Rright . When the switch is opened,
the equivalent resistance of the circuit increases from
the battery decreases, from
(a)

3
2

3
2

R + r to 2 R + r. Thus the current delivered by

to
. Note that this is LESS than a 50% decrease.
2R + r
R+r

Because the current from the battery has decreased, the voltage drop across Rleft will decrease,
since it will have less current than before. The voltage drop across Rright decreases to 0, since no
current is flowing in it. The voltage drop across Rmiddle will increase, because even though the
total current has decreased, the current flowing through Rmiddle has increased since before the
switch was opened, only half the total current was flowing through Rmiddle .
Vleft decreases; Vmiddle increases; Vright goes to 0

(b)

By Ohms law, the current is proportional to the voltage for a fixed resistance.
I left decreases; I middle increases; I right goes to 0

(c)

Since the current from the battery has decreased, the voltage drop across r will decrease, and
thus the terminal voltage increases.

(d)

With the switch closed, the equivalent resistance is


I closed =

3
2

3
2

R + r. Thus the current in the circuit is

, and the terminal voltage is given by Eq. 191.


R+r

Vterminal = I closed r =
closed

3
2

r
0.50
r = 1 3
= (9.0 V) 1 3

R+r
R + r
(5.50 ) + 0.50

2
2

= 8.486 V 8.5 V

Copyright 2014 Pearson Education, Inc. All rights reserved. This material is protected under all copyright laws as they currently exist.
No portion of this material may be reproduced, in any form or by any means, without permission in writing from the publisher.

DC Circuits

(e)

19-13

With the switch open, the equivalent resistance is 2 R + r. Thus the current in the circuit is

I closed =
, and again the terminal voltage is given by Eq. 191.
2R + r
Vterminal = I closed r =
closed

r
0.50

r = 1

= (9.0 V) 1
+
.

+
.

2R + r
2
R
r
2(5
50
)
0
50

= 8.609 V 8.6 V

23.

Find the maximum current and resulting voltage for each resistor under the power restriction.
P = I 2R =
I1400 =
I 2500 =
I 3700 =

V2
R

I=

0.5 W
1.4 103
0.5 W
2.5 103
0.5 W
3.7 103

P
, V = RP
R

= 0.0189 A V1800 = (0.5 W)(1.4 103 ) = 26.5 V


= 0.0141 A V2500 = (0.5 W)(2.8 103 ) = 35.4 V
= 0.0116 A V3700 = (0.5 W)(3.7 103 ) = 43.0 V

The parallel resistors have to have the same voltage, so the voltage across that combination is limited
to 35.4 V. That would require a current given by Ohms law and the parallel combination of the two
resistors.
I parallel =

Vparallel
Rparallel

1
1
1
1
= Vparallel
+
+
= (35.4 V)
= 0.0237 A
2500 3700
R2500 R3700

This is more than the maximum current that can be in R1400 . Thus the maximum current that R1400
can carry, 0.0189 A, is the maximum current for the circuit. The maximum voltage that can be applied
across the combination is the maximum current times the equivalent resistance. The equivalent
resistance is the parallel combination of R2500 and R3700 added to R1400 .
Vmax

1
1

1
1
1
1

= I max Req = I max R1800


+
=
.

+
(0 0189 A) 1400


R2800 R3700
2500 3700

= 54.66 V 55 V

24.

(a)

Note that adding resistors in series always results in a larger resistance, and adding resistors in
parallel always results in a smaller resistance. Closing the switch adds another resistor in parallel
with R3 and R4 , which lowers the net resistance of the parallel portion of the circuit and thus
lowers the equivalent resistance of the circuit. That means that more current will be delivered by
the battery. Since R1 is in series with the battery, its voltage will increase. Because of that
increase, the voltage across R3 and R4 must decrease so that the total voltage drops around the
loop are equal to the battery voltage. Since there was no voltage across R2 until the switch was
closed, its voltage will increase. To summarize:
V1 and V2 increase; V3 and V4 decrease

Copyright 2014 Pearson Education, Inc. All rights reserved. This material is protected under all copyright laws as they currently exist.
No portion of this material may be reproduced, in any form or by any means, without permission in writing from the publisher.

19-14

Chapter 19

(b)

By Ohms law, the current is proportional to the voltage for a fixed resistance. Thus
I1 and I 2 increase; I 3 and I 4 decrease

(c)

Since the battery voltage does not change and the current delivered by the battery increases, the
power delivered by the battery, found by multiplying the voltage of the battery by the current
delivered, increases .

(d)

Before the switch is closed, the equivalent resistance is R3 and R4 in parallel, combined with
R1 in series.
1
1
+
Req = R1 +

R3 R4

2
= 155 +

155

= 232.5

The current delivered by the battery is the same as the current through R1.
I total =

Vbattery
Req

22.0 V
= 0.09462 A = I1
232.5

The voltage across R1 is found by Ohms law.


V1 = IR1 = (0.09462 A)(155 ) = 14.666 V

The voltage across the parallel resistors is the battery voltage less the voltage across R1.
Vp = Vbattery V1 = 22.0 V 14.666 V = 7.334 V

The current through each of the parallel resistors is found from Ohms law.
I3 =

Vp
R2

7.334 V
= 0.04732 A = I 4
155

Notice that the current through each of the parallel resistors is half of the total current, within the
limits of significant figures. The currents before closing the switch are as follows.
I1 = 0.0946 A

I 3 = I 4 = 0.0473 A

After the switch is closed, the equivalent resistance is R2 , R3 , and R4 in parallel, combined with
R1 in series. Do a similar analysis.
Req

1
1
1
= R1 +
+
+

R
R
R
3
4
2

I total =

Vbattery
Req

3
= 155 +

155

= 206.7

22.0 V
= 0.1064 A = I1 V1 = IR1 = (0.1064 A)(155 ) = 16.49 V
206.7

Vp = Vbattery V1 = 22.0 V 16.49 V = 5.51 V I 2 =

Vp
R2

5.51 V
= 0.0355 A = I 3 = I 4
155

Notice that the current through each of the parallel resistors is one-third of the total current,
within the limits of significant figures. The currents after closing the switch are as follows.
I1 = 0.106 A

I 2 = I3 = I 4 = 0.0355 A

Yes, the predictions made in part (b) are all confirmed.

Copyright 2014 Pearson Education, Inc. All rights reserved. This material is protected under all copyright laws as they currently exist.
No portion of this material may be reproduced, in any form or by any means, without permission in writing from the publisher.

DC Circuits

25.

19-15

All of the resistors are in series, so the equivalent resistance is just the sum of the resistors. Use Ohms
law to find the current, and show all voltage changes starting at the negative pole of the battery and
going counterclockwise.
I=

9.0 V
=
= 0.3529 A 0.35 A
Req (9.5 + 14.0 + 2.0)

voltages = 9.0 V (9.5 )(0.3529 A) (14.0 )(0.3529 A) (2.0 )(0.3529 A)


= 9.0 V 3.35 V 4.94 V 0.71 V = 0

26.

Apply Kirchhoffs loop rule to the circuit, starting at the upper left corner of the circuit diagram, in
order to calculate the current. Assume that the current is flowing clockwise.
I (2.0 ) + 18 V I (4.8 ) 12 V I (1.0 ) = 0 I =

6V
= 0.769 A
7.8

The terminal voltage for each battery is found by summing the potential differences across the internal
resistance and emf from left to right. Note that for the 12-V battery, there is a voltage gain going across
the internal resistance from left to right.
18-V battery: Vterminal = I (2.0 ) + 18 V = (0.769 A)(2.0 ) + 18 V = 16.46 V 16 V
12-V battery: Vterminal = I (1.0 ) + 12 V = (0.769 A)(1.0 ) + 12 V = 12.769 V 13 V

27.

To find the potential difference between points a and b, the current must be found from Kirchhoffs
loop law. Start at point a and go counterclockwise around the entire circuit, taking the current to be
counterclockwise.
IR + IR IR + IR = 0 I =

2R

Vab = Va Vb = IR + IR = 2 IR = 2

R= 0
2R

Notice that the actual values for the battery voltages and the resistances were not used.
28.

(a)

There are three currents involved, so there must be three independent


equations to determine those three currents. One comes from
Kirchhoffs junction rule applied to the junction of the three branches
on the right of the circuit.
I 2 = I1 + I 3

I1

I1 = I 2 I 3

Another equation comes from Kirchhoffs loop rule applied to the top
loop, starting at the negative terminal of the battery and progressing
clockwise.
1 I1 R1 I 2 R2 = 0 9 = 25I1 + 68I 2

The final equation comes from Kirchhoffs loop rule applied to the
bottom loop, starting at the negative terminal of the battery and
progressing counterclockwise.

R1

R2
I2
R3

2
I3

2 I 3 R3 I 2 R2 = 0 12 = 35I 3 + 68I 2

Substitute I1 = I 2 I 3 into the top loop equation, so that there are two equations with two
unknowns.
9 = 25 I1 + 68 I 2 = 25( I 2 I 3 ) + 68 I 2 = 93I 2 25 I 3 ; 12 = 35 I 3 + 68I 2

Copyright 2014 Pearson Education, Inc. All rights reserved. This material is protected under all copyright laws as they currently exist.
No portion of this material may be reproduced, in any form or by any means, without permission in writing from the publisher.

19-16

Chapter 19

Solve the bottom loop equation for I 2 and substitute into the top loop equation, resulting in an
equation with only one unknown, which can be solved.
12 = 35 I 3 + 68 I 2

I2 =

12 35 I 3
68

12 35I 3
9 = 93I 2 25 I 3 = 93
25I 3
68

I3 =

612 = 1116 3255I 3 1700 I 3

12 35I 3
504
= 0.1017 A 0.10 A, up ; I 2 =
= 0.1241 A 0.12 A, left
4955
68

I1 = I 2 I 3 = 0.0224 A 0.02 A, right

(b)

We can include the internal resistances simply by adding 1.0 to R1 and R3 . So let R1 = 26
and let R3 = 36 . Now re-work the problem exactly as in part (a).
I 2 = I1 + I 3

I1 = I 2 I 3

1 I1 R1 I 2 R2 = 0 9 = 26 I1 + 68 I 2
2 I 3 R3 I 2 R2 = 0 12 = 36 I 3 + 68 I 2
9 = 26 I1 + 68 I 2 = 26( I 2 I 3 ) + 68I 2 = 94 I 2 26 I 3 ; 12 = 36 I 3 + 68I 2

12 = 36 I 3 + 68I 2

I2 =

12 36 I 3 3 9 I 3
=
68
17

3 9 I3
9 = 94 I 2 26 I 3 = 94
26 I 3 153 = 282 846 I 3 442 I 3
17
3 12 I 3
129
I3 =
= 0.1002 A 0.10 A, up ; I 2 =
= 0.1234 A 0.12 A, left
1288
17
I1 = I 2 I 3 = 0.0232 A 0.02 A, right

The currents are unchanged to the nearest 0.01 A by the inclusion of the internal resistances.
29.

This circuit is identical to Example 198 and Fig. 1913 except for the numeric values. So we may
copy the equations developed in that Example but use the current values.
Eq. (i): I 3 = I1 + I 2

Eq. (ii): 34 I1 + 45 48I 3 = 0

Eq. (iii): 34 I1 + 19 I 2 85 = 0

Eq. (iv): I 2 =

85 + 34 I1
= 4.474 + 1.789 I1
19

45 34 I1
= 0.938 0.708 I1
48
I 3 = I1 + I 2 0.938 0.708 I1 = I1 + 4.474 + 1.789 I1 I1 = 1.011 A

Eq. (v): I 3 =

I 2 = 4.474 + 1.789 I1 = 2.665 A; I 3 = 0.938 0.708 I1 = 1.654 A

(a)

To find the potential difference between points a and d, start at point a and add each individual
potential difference until reaching point d. The simplest way to do this is along the top branch.
Vad = Vd Va = I1 (34 ) = (1.011 A)(34 ) = 34.37 V 34 V
Slight differences might be obtained in the final answer depending on the branch used, due to
rounding. For example, using the bottom branch, we get the following.
Vad = Vd Va = 1 I 2 (19 ) = 85 V (2.665 A)(19 ) = 34.365 V 34 V

Copyright 2014 Pearson Education, Inc. All rights reserved. This material is protected under all copyright laws as they currently exist.
No portion of this material may be reproduced, in any form or by any means, without permission in writing from the publisher.

DC Circuits

(b)

19-17

For the 85-V battery, the terminal voltage is the potential difference from point g to point e. For
the 45-V battery, the terminal voltage is the potential difference from point d to point b.
85 = V battery: Vterminal = 1 I 2 r = 85 V (2.665 A)(1.0 ) = 82 V
45 = V battery: Vterminal = 2 I 3 r = 45 V (1.654 A)(1.0 ) = 43 V

30.

I1

There are three currents involved, so there must be three


independent equations to determine those three currents. One
comes from Kirchhoffs junction rule applied to the junction
of the three branches at the top center of the circuit.

58V
120

I1 = I 2 + I 3

I3

25

74

3.0V

I2
56

110

Another equation comes from Kirchhoffs loop rule applied to


the left loop, starting at the negative terminal of the battery and progressing counterclockwise.
58 V I1 (120 ) I1 (74 ) I 2 (56 ) = 0 58 = 194 I1 + 56 I 2

The final equation comes from Kirchhoffs loop rule applied to the right loop, starting at the negative
terminal of the battery and progressing counterclockwise.
3.0 V I 3 (25 ) + I 2 (56 ) I 3 (110 ) = 0 3 = 56 I 2 + 135I 3

Substitute I1 = I 2 + I 3 into the left loop equation, so that there are two equations with two unknowns.
58 = 194( I 2 + I 3 ) + 56 I 2 = 250 I 2 + 194 I 3

Solve the right loop equation for I 2 and substitute into the left loop equation, resulting in an equation
with only one unknown, which can be solved.
3 = 56 I 2 + 135 I 3

I2 =

135 I 3 3
135 I 3 3
; 58 = 250 I 2 + 194 I 3 = 250
+ 194 I 3
56
56

3998 = 44614 I 3
I 3 = 0.08961 A;

I2 =

135 I3 3
= 0.1625 A; I1 = I 2 + I 3 = 0.2521 A
56

The current in each resistor is as follows:


120 : 0.25 A
31.

74 : 0.25 A

56 : 0.16 A

25 : 0.090 A

110 : 0.090 A

Because there are no resistors in the bottom branch, it is possible to write Kirchhoff loop equations that
only have one current term, making them easier to solve. To find the current through R1 , go around
the outer loop counterclockwise, starting at the lower left corner.
V3 I1 R1 + V1 = 0 I1 =

V3 + V1 6.0 V + 9.0 V
=
= 0.68 A, left
R1
22

Copyright 2014 Pearson Education, Inc. All rights reserved. This material is protected under all copyright laws as they currently exist.
No portion of this material may be reproduced, in any form or by any means, without permission in writing from the publisher.

19-18

Chapter 19

To find the current through R2 , go around the lower loop counterclockwise, starting at the lower left
corner.
V3 I 2 R2 = 0 I 2 =

32.

V3 6.0 V
=
= 0.33 A, left
R2 18

There are three currents involved, so there must be three


independent equations to determine those three currents. One
comes from Kirchhoffs junction rule applied to the junction of
the three branches on the left of the circuit.
I1 = I 2 + I 3

1.4

22

I2
I3

Another equation comes from Kirchhoffs loop rule applied to


the outer loop, starting at the lower left corner and progressing
counterclockwise.

I1

9.0V

18
6.0V

1.4

I 3 (1.4 ) + 6.0 V I1 (22 ) I1 (1.4 ) + 9.0 V = 0


15 = 23.4 I1 + 1.4 I 3
The final equation comes from Kirchhoffs loop rule applied to the bottom loop, starting at the lower
left corner and progressing counterclockwise.
I 3 (1.4 ) + 6.0 V + I 2 (18 ) = 0 6 = 18 I 2 + 1.4 I 3

Substitute I1 = I 2 + I 3 into the top loop equation, so that there are two equations with two unknowns.
15 = 23.4 I1 + 1.4 I 3 = 23.4( I 2 + I 3 ) + 1.4 I 3 = 23.4 I 2 + 24.8 I 3 ; 6 = 18I 2 + 1.4 I 3

Solve the bottom loop equation for I 2 and substitute into the top loop equation, resulting in an
equation with only one unknown, which can be solved.
6 + 1.4 I 3
18
6 + 1.4 I 3
15 = 23.4 I 2 + 24.8I 3 = 23.4
+ 24.8 I 3 270 = 140.4 + 32.76 I 3 + 446.4 I 3
18

6 = 18I 2 + 1.4 I 3 I 2 =

6 + 1.4 I 3 6 + 1.4(0.8565)
410.4
= 0.8565 A ; I 2 =
=
= 0.2667 A 0.27 A, left
479.16
18
18
I1 = I 2 + I 3 = 0.5898 A 0.59 A, left
I3 =

33.

(a)

We label each of the currents as shown in the


accompanying figure. Using Kirchhoffs junction rule
and the first three junctions (ac), we write equations
relating the entering and exiting currents.
I = I1 + I 2

[1]

I 2 = I3 + I 4

[2]

I1 + I 4 = I5

[3]

Copyright 2014 Pearson Education, Inc. All rights reserved. This material is protected under all copyright laws as they currently exist.
No portion of this material may be reproduced, in any form or by any means, without permission in writing from the publisher.

DC Circuits

19-19

We use Kirchhoffs loop rule to write equations for loops abca, abcda, and bdcb.
0 = I 2 R I 4 R + I1 R

[4]

0 = I 2 R I3 R +

[5]

0 = I3 R + I5 R + I 4 R

[6]

We have six unknown currents and six equations. We solve these equations by substitution.
First, insert Eq. [3] into [6] to eliminate current I5 . Next insert Eq. [2] into Eqs. [1], [4], and [5]
to eliminate I 2 .
0 = I 3 R + ( I1 + I 4 ) R + I 4 R 0 = I 3 R + I1 R + 2 I 4 R

[6]

I = I1 + I 3 + I 4

[1]

0 = ( I 3 + I 4 ) R I 4 R + I1 R 0 = I 3 R 2 I 4 R + I1 R

[4]

0 = ( I 3 + I 4 ) R I 3 R + 0 = I 4 R 2 I 3 R +

[5]

Next we solve Eq. [4] for I 4 and insert the result into Eqs. [1], [5], and [6].
0 = I 3 R 2 I 4 R + I1 R I 4 = 12 I1 12 I 3
I = I1 + I 3 + 12 I1 12 I 3

I = 32 I1 + 12 I 3

[1]

0 = I 3 R + I1 R + 2( 12 I1 12 I 3 ) R = 2 I 3 R + 2 I1R I1 I 3

[6]

0 = ( 12 I1 12 I 3 ) R 2 I 3 R + 0 = 12 I1 R 32 I 3 R +

[5]

Finally we substitute Eq. [6] into Eq. [5] and solve for I1. We insert this result into Eq. [1] to
write an equation for the current through the battery in terms of the battery emf and resistance.
0 = 12 I1 R 32 I1 R + I1 =

(b)

(a)

I=

We divide the battery emf by the current to determine the effective resistance.
Req =

34.

; I = 32 I1 + 12 I1 = 2 I1
2R

=
= R
I /R

Since there are three currents to determine, there must be three


independent equations to determine those three currents. One
comes from Kirchhoffs junction rule applied to the junction
near the negative terminal of the middle battery.
I1 = I 2 + I 3

Another equation comes from Kirchhoffs loop rule applied to


the top loop, starting at the negative terminal of the middle
battery and progressing counterclockwise. We add series
resistances.
12.0 V I 2 (12 ) + 12.0 V I1 (35 ) = 0 24 = 35 I1 + 12 I 2

The final equation comes from Kirchhoffs loop rule applied to the bottom loop, starting at the
negative terminal of the middle battery and progressing clockwise.
12.0 V I 2 (12 ) 6.0 V + I 3 (28 ) = 0 6 = 12 I 2 28I 3

Copyright 2014 Pearson Education, Inc. All rights reserved. This material is protected under all copyright laws as they currently exist.
No portion of this material may be reproduced, in any form or by any means, without permission in writing from the publisher.

19-20

Chapter 19

Substitute I1 = I 2 + I 3 into the top loop equation, so that there are two equations with two
unknowns.
24 = 35 I1 + 12 I 2 = 35( I 2 + I 3 ) + 12 I 2 = 47 I 2 + 35 I 3

Solve the bottom loop equation for I 2 and substitute into the top loop equation, resulting in an
equation with only one unknown, which can be solved for I 3 .
6 + 28 I 3 3 + 14 I 3
3 + 14 I 3
; 24 = 47 I 2 + 35I 3 = 47
=
+ 35 I 3
12
6
6

6 + 28I 3
I 3 = 3.46 mA ; I 2 =
= 0.508 A ; I1 = I 2 + I3 = 0.511 A
12

6 = 12 I 2 28I 3

(b)

I2 =

The terminal voltage of the 6.0-V battery is 6.0 V I 3 r = 6.0 V (3.46 103 A)(1.0 )
= 5.997 V 6.0 V .

35.

This problem is the same as Problem 34, except the total resistance in the top branch is now 23
instead of 35 . We simply reproduce the adjusted equations here without the prose.
I1 = I 2 + I 3
12.0 V I 2 (12 ) + 12.0 V I1 (23 ) = 0 24 = 23I1 + 12 I 2
12.0 V I 2 (12 ) 6.0 V + I 3 (28 ) = 0 6 = 12 I 2 28 I 3
24 = 23I1 + 12 I 2 = 23( I 2 + I 3 ) + 12 I 2 = 35 I 2 + 23I3
6 = 12 I 2 28 I 3

I2 =

6 + 28I 3 3 + 14 I 3
3 + 14 I 3
; 24 = 35I 2 + 23I 3 = 35
=
+ 23I 3
12
6
6

39 = 628 I 3
I 3 = 0.0621 A; I 2 =

36.

6 + 28 I 3
= 0.6449 A; I1 = I 2 + I 3 = 0.707 A 0.71 A
12

Define I1 to be the current to the right through the 2.00-V


battery ( 1 ), and I 2 to be the current to the right through the
3.00 V battery ( 2 ). At the junction, they combine to give
current I = I1 + I 2 to the left through the top branch. Apply
Kirchhoffs loop rule, first to the upper loop and then to the
outer loop, and solve for the currents.
1 I1r ( I1 + I 2 ) R = 0 1 ( R + r ) I1 RI 2 = 0

I1 + I 2

r
I1
r

I2

2 I 2 r ( I1 + I 2 ) R = 0 2 RI1 ( R + r ) I 2 = 0

Solve the first equation for I 2 and substitute into the second equation to solve for I1.
1 ( R + r ) I1 2.00 4.350 I1
=
= 0.500 1.0875I1
R
4.00
2 RI1 ( R + r ) I 2 = 3.00 4.00 I1 (4.35)(0.500 1.0875I1 ) = 0
1 ( R + r ) I1 RI 2 = 0 I 2 =
0.825 = 0.7306 I1
I1 = 1.129 A; I 2 = 0.500 1.0875 I1 = 1.728 A

Copyright 2014 Pearson Education, Inc. All rights reserved. This material is protected under all copyright laws as they currently exist.
No portion of this material may be reproduced, in any form or by any means, without permission in writing from the publisher.

DC Circuits

19-21

The voltage across R is its resistance times I = I1 + I 2 .


VR = R ( I1 + I 2 ) = (4.00 )(1.129 A + 1.728 A) = 2.396 V 2.40 V

Note that the top battery is being chargedthe current is flowing through it from positive to negative.
37.

Take 100 of the batteries and connect them in series, which would give a total voltage of 300 volts. Do
that again with the next 100 batteries, and again with the last 100 batteries. That gives three sets, each
with a total voltage of 300 volts. Then those three sets can be connected in parallel with each other.
The total combination of batteries would have a potential difference of 300 volts.
Another possibility is to connect three batteries in parallel, which would provide a potential difference
of 3 volts. Make 100 sets of those three-battery combinations and connect those 100 sets in series.
That total combination would also have a potential difference of 300 volts.

38.

(a)

Capacitors in parallel add according to Eq. 195.


Ceq = C1 + C2 + C3 + C4 + C5 + C6 = 6(4.8 106 F) = 2.88 105 F = 28.8 F

(b)

Capacitors in series add according to Eq. 196.

Ceq

39.

1
1
1
1
1
1
= +
+
+
+
+

C1 C2 C3 C4 C5 C6

6
=
4.8 106 F

4.8 106 F
= 8.0 107 F
6

The series capacitors add reciprocally, and then the parallel combination is found by adding linearly.
1
1
Ceq = +

C1 C2

40.

(a)

1
1
+ C3 =
+

6
6
3.00 10 F 4.00 10 F

+ 2.00 106 F = 3.71 106 F = 3.71 F

The full voltage is across the 2.00- F capacitor, so V3 = 21.0 V . To find the voltage cross the
two capacitors in series, find their equivalent capacitance and the charge stored. That charge will
be the same for both of the series capacitors. Finally, use that charge to determine the voltage on
each capacitor. The sum of the voltages across the series capacitors is 26.0 V.
1
1
Ceq = +

C
C
2
1

1
1
=
+

6
6

3.00 10 F 4.00 10 F

= 1.714 106 F

Qeq = CeqV = (1.714 106 F)(21.0 V) = 3.599 105 C


V1 =

(b)

Qeq
C

3.599 105 C
3.00 106 F

= 12.0 V

V2 =

Qeq
C

3.599 105 C
4.00 106 F

= 9.0 V

We have found two of the three charges already. The charge Qeq = 3.599 105 C is the charge
on the two capacitors in series. The other charge is found by using the full voltage.
Q3 = C3V = (2.00 106 F)(21.0 V) = 4.20 105 C
Q1 = Q2 = 3.60 105 C; Q3 = 4.20 105 C

Copyright 2014 Pearson Education, Inc. All rights reserved. This material is protected under all copyright laws as they currently exist.
No portion of this material may be reproduced, in any form or by any means, without permission in writing from the publisher.

19-22

41.

Chapter 19

To reduce the net capacitance, another capacitor must be added in series.


1
1
1
=
+
Ceq C1 C2
C2 =

C1Ceq
C1 Ceq

1
1
1 C1 Ceq
=

=
C2 Ceq C1
C1Ceq
(2.9 109 F)(1.2 109 F)
(2.9 10

F) (1.2 10

F)

= 2.047 109 F 2.0 nF

Yes, an existing connection needs to be broken in the process. One of the connections of the original
capacitor to the circuit must be disconnected in order to connect the additional capacitor in series.
42.

Capacitors in parallel add linearly, so adding a capacitor in parallel will increase the net capacitance
without removing the 5.0- F capacitor.
7.0 F + C = 16 F C = 9 F connected in parallel

43.

The maximum capacitance is found by connecting the capacitors in parallel.


Cmax = C1 + C2 + C3 = 3.2 109 F + 5.8 109 F + 1.00 108 F = 1.90 108 F in parallel

The minimum capacitance is found by connecting the capacitors in series.


Cmin

44.

1
1
1
= +
+

C1 C2 C3

1
1
1
=
+
+
3.2 109 F 5.8 109 F 1.00 108 F

= 1.7 109 F in series

From the diagram, we see that C1 and C2 are in parallel and


C3 and C4 are in parallel. Those two combinations are then in
series with each other. Use those combinations to find the
equivalent capacitance. We use subscripts to indicate which
capacitors have been combined.
C12 = C1 + C2 ; C34 = C3 + C4 ;
1
C1234

C1234 =

1
1
1
1
+
=
+
C12 C34 C1 + C2 C3 + C4

(C1 + C2 )(C3 + C4 )
(C1 + C2 + C3 + C4 )

45.

The voltage across C1 is the full 10 V from the battery. Since the other two capacitors are identical,
they will each have 5 V across them so that the total voltage across the two of them is 10 V. Thus
V1 /V2 = 10 V/5 V = 2/1.

46.

When the capacitors are connected in series, they each have the same charge as the net capacitance.
1

(a)

Q1 = Q2 = Qeq

1
1
1
1
= CeqV = +
+
(9.0 V)
V =

6
C1 C2
0.50 10 F 1.40 10 F

= 3.316 106 C
V1 =

Q1 3.316 106 C
=
= 6.632 V 6.6 V
C1
0.50 106 F

V2 =

Q2 3.316 106 C
=
= 2.369 V 2.4 V
C2
1.4 106 F

Copyright 2014 Pearson Education, Inc. All rights reserved. This material is protected under all copyright laws as they currently exist.
No portion of this material may be reproduced, in any form or by any means, without permission in writing from the publisher.

DC Circuits

(b)

19-23

Q1 = Q2 = Qeq = 3.316 106 C 3.3 106 C

When the capacitors are connected in parallel, they each have the full potential difference.
(c)

V1 = 9.0 V

V2 = 9.0 V

Q1 = C1V1 = (0.50 106 F)(9.0 V) = 4.5 106 C

Q2 = C2V2 = (1.4 106 F)(9.0 V) = 1.3 105 C


47.

The energy stored by a capacitor is given by Eq. 1710, PE = 12 CV 2 .


= 4PEinitial

PE final

1C
V2
2 final final

2
= 4 12 CinitialVinitial

One simple way to accomplish this is to have Cfinal = 4Cinitial and Vfinal = Vinitial . In order to keep the
voltage the same for both configurations, any additional capacitors must be connected in parallel to the
original capacitor. In order to multiply the capacitance by a factor of 4, we recognize that capacitors
added in parallel add linearly. Thus if a capacitor of value 3C = 750 pF were connected in parallel to
the original capacitor, then the final capacitance would be 4 times the original capacitance with the
same voltage, so the potential energy would increase by a factor of 4.
48.

The capacitors are in parallel, so the potential is the same for each capacitor, and the total charge on
the capacitors is the sum of the individual charges. We use Eqs. 177 and 178.
Q1 = C1V = 0

A
A1
A
V ; Q2 = C2V = 0 2 V ; Q3 = C3V = 0 3 V
d1
d2
d3

Qtotal = Q1 + Q2 + Q3 = 0

Cnet =

49.

Qtotal
V

A
A
A
A1
A
A
V + 0 2 V + 0 3 V = 0 1 + 0 2 + 0 3 V
d1
d2
d3
d2
d3
d1

A1
A
A
+ 0 2 + 0 3 V
0
d1
d2
d3
A
A
A
=
= 0 1 + 0 2 + 0 3 = C1 + C2 + C3
V
d2
d3
d1

We have CP = C1 + C2 = 35.0 F and

1
1
1
1
. Solve for C1 and C2 in terms of CP
=
+
=
CS C1 C2 4.8 F

and CS .
(C C1 ) + C1
CP
1
1
1
1
1
=
+
=
+
= P
=
CS C1 C2 C1 CP C1
C1 (CP C1 )
C1 (CP C1 )
CP
1
=
CS C1 (CP C1 )
C1 =

C12 CP C1 + CP CS = 0

CP CP2 4CP CS

2
= 29.3 F, 5.7 F

35.0 F (35.0 F) 2 4(35.0 F)(4.8 F)


2

C2 = CP C1 = 35.0 F 29.3 F = 5.7 F or 35.0 F 5.7 F = 29.3 F

So the two values are 29.3 F and 5.7 F .

Copyright 2014 Pearson Education, Inc. All rights reserved. This material is protected under all copyright laws as they currently exist.
No portion of this material may be reproduced, in any form or by any means, without permission in writing from the publisher.

19-24

50.

Chapter 19

We want a small voltage drop across C1. Since V = Q /C , if we put


the smallest capacitor in series with the battery, there will be a large
C3
voltage drop across it. Then put the two larger capacitors in parallel,
C1
so that their equivalent capacitance is large and therefore will have a
small voltage drop across them. So put C1 and C3 in parallel with
V0
C2
each other, and then put that combination in series with C2 . See the
diagram. To calculate the voltage across C1 , find the equivalent
capacitance and the net charge. That charge is used to find the voltage drop across C2 , and then that
voltage is subtracted from the battery voltage to find the voltage across the parallel combination.
C + C2 + C3
1
1
1
=
+
= 1
Ceq C2 C1 + C3 C2 (C1 + C3 )
V1 = V0 V2 = V0

Qeq
C2

= V0

Ceq =

CeqV0
C2

Qeq
C2 (C1 + C3 )
Q
; Qeq = CeqV0 ; V2 = 2 =
;
C1 + C2 + C3
C2 C2

C2 (C1 + C3 )
V0
C1 + C2 + C3
C2
1.5 F
V0 =
= V0
=
(12 V)
C2
C1 + C2 + C3
6.5 F

= 2.8 V

51.

(a)

From the diagram, we see that C1 and C2 are in series. That combination is in parallel with C3 ,
and then that combination is in series with C4 . Use those combinations to find the equivalent
capacitance. We use subscripts to indicate which capacitors have been combined.
1
1 1
= +
C12 C C
1
C1234

(b)

C12 = 12 C ; C123 = C12 + C3 = 12 C + C = 32 C

1
1
2 1
5
+
=
+ =
C123 C4 3C C 3C

C1234 =

3C
5

The charge on the equivalent capacitor C1234 is given by Q1234 = C1234V = 53 CV . This is the
charge on both of the series components of C1234 .
Q123 = 53 CV = C123V123 = 32 CV123
Q4 = 53 CV = C4V4

V123 = 52 V

V4 = 53 V

The voltage across the equivalent capacitor C123 is the voltage across both of its parallel
components. Note that the sum of the charges across the two parallel components of C123 is the
same as the total charge on the two components,

3 CV .
5

Copyright 2014 Pearson Education, Inc. All rights reserved. This material is protected under all copyright laws as they currently exist.
No portion of this material may be reproduced, in any form or by any means, without permission in writing from the publisher.

DC Circuits

19-25

( 12 C ) ( 52 V ) = 15 CV
V123 = 52 V = V3 ; Q3 = C3V3 = C ( 52 V ) = 52 CV
V123 = 52 V = V12 ; Q12 = C12V12 =

Finally, the charge on the equivalent capacitor C12 is the charge on both of the series
components of C12 .
Q12 = 15 CV = Q1 = C1V1 V1 = 15 V ; Q12 = 15 CV = Q2 = C1V2

V2 = 15 V

Here are all the results, gathered together.


Q1 = Q2 = 15 CV ; Q3 = 52 CV ; Q4 = 53 CV
V1 = V2 = 15 V ; V3 = 52 V ; V4 = 53 V

52.

We take each of the given times as the time constant of the RC combination.

= RC R =

R1s =

R4s =
R15s =

1s
1 10

= 1 106 ; R2s =

4s
1 10

15 s
1 106 F

= 4 106 ; R2s =

2s
1 106 F

= 2 106 ;

8s
1 10

= 8 106 ;

= 15 106

So we estimate the range of resistance to be 1 M 15 M .


53.

From Eq. 197, the product RC is equal to the time constant.

= RC R =
54.

(a)

3.0 106 F

= 1.0 106

From Eq. 197, the product RC is equal to the time constant.

= RC C =
(b)

3.0 s

18.0 106 s
3

15.0 10

= 1.20 109 F

Since the battery has an EMF of 24.0 V, if the voltage across the resistor is 16.0 V, then the
voltage across the capacitor will be 8.0 V as it charges. Use the expression for the voltage across
a charging capacitor.
t
V
V
VC = (1 et / ) et / = 1 C = ln 1 C

V
t = ln 1 C

16.0 V

6
5
= (18.0 10 s)ln 1 24.0 V = 1.98 10 s

Copyright 2014 Pearson Education, Inc. All rights reserved. This material is protected under all copyright laws as they currently exist.
No portion of this material may be reproduced, in any form or by any means, without permission in writing from the publisher.

19-26

55.

Chapter 19

The current for a capacitor-charging circuit is given by Eq. 197d,


with R the equivalent series resistance and C the equivalent series
capacitance.
I=

(t /Req Ceq )
e
Req

R1

R2 C2

+
S

C1

+
+

IReq
C1C2 I (R1 + R2 )
t = Req Ceq ln
= (R1 + R2 )
ln

C1 + C2

(3.8 106 F) 2 (1.50 103 A)(4400 )
3
= (4400 )
ln
= 7.4 10 s
6
(16.0V)
7.6 10 F

56.

The voltage of the discharging capacitor is given by VC = V0 et /RC . The capacitor voltage is to be
0.0025 V0 .
VC = V0 et /RC

0.0010 V0 = V0 et /RC

0.0010 = et /RC

ln (0.010) =

t
RC

t = RC ln (0.010) = (8.7 103 )(3.0 106 F) ln (0.0025) = 0.16 s

57.

(a)

At t = 0, the capacitor is uncharged, so there is no voltage difference across it. The capacitor is a
short, so a simpler circuit can be drawn just by eliminating the capacitor. In this simpler
circuit, the two resistors on the right are in parallel with each other, and then in series with the
resistor by the switch. The current through the resistor by the switch splits equally when it
reaches the junction of the equal, parallel resistors.
1 1
Req = R + +
R R

I1 =

(b)

= 32 R

2
=
=
; I 2 = I 3 = 12 I1 =
Req 32 R 3R
3R

At t = , the capacitor will be fully charged and there will be no current


in the branch containing the capacitor, so a simpler circuit can be drawn
by eliminating that branch. In this simpler circuit, the two resistors are in
series, and they both have the same current.
Req = R + R = 2 R I1 = I 2 =

(c)

=
; I3 = 0
Req
2R

At t = , since there is no current through the branch containing the capacitor, there is no
potential drop across that resistor. Therefore, the voltage difference across the capacitor equals
the voltage difference across the resistor through which I 2 flows.

VC = VR2 = I 2 R =
R =
2R

1
2

Copyright 2014 Pearson Education, Inc. All rights reserved. This material is protected under all copyright laws as they currently exist.
No portion of this material may be reproduced, in any form or by any means, without permission in writing from the publisher.

DC Circuits

58.

(a)

19-27

With the switch open, the resistors are in series with each other, so have the same current. Apply
the loop rule clockwise around the left loop, starting at the negative terminal of the source, to
find the current.
V IR1 IR2 = 0 I =

V
24 V
=
= 1.818 A
R1 + R2 8.8 + 4.4

The voltage at point a is the voltage across the 4.4 -resistor.


Va = IR2 = (1.818 A)(4.4 ) = 8.0 V

(b)

With the switch open, the capacitors are in series with each other. Find the equivalent
capacitance. The charge stored on the equivalent capacitance is the same value as the charge
stored on each capacitor in series.
1
1
1
=
+
Ceq C1 C2

Ceq =

C1C2
(0.48 F)(0.36 F)
=
= 0.2057 F
C1 + C2 (0.48 F + 0.36 F)

Qeq = VCeq = (24.0 V)(0.2057 F) = 4.937 C = Q1 = Q2

The voltage at point b is the voltage across the 0.24- F capacitor.


Vb =

(c)

(d)

Q2 4.937 C
=
= 13.7 V 14 V
C2
0.36 F

The switch is now closed. After equilibrium has been reached for a long time, there is no current
flowing in the capacitors, so the resistors are again in series, and the voltage of point a must be
8.0 V. Point b is connected by a conductor to point a, so point b must be at the same potential as
point a, 8.0 V . This also means that the voltage across C2 is 8.0 V, and the voltage across C1
is 16 V.
Find the charge on each of the capacitors, which are no longer in series.
Q1 = V1C1 = (16 V)(0.48 F) = 7.68 C
Q2 = V2C2 = (8.0 V)(0.36 F) = 2.88 C

When the switch was open, point b had a net charge of 0, because the charge on the negative
plate of C1 had the same magnitude as the charge on the positive plate of C2 . With the switch
closed, these charges are not equal. The net charge at point b is the sum of the charge on the
negative plate of C1 and the charge on the positive plate of C2 .
Qb = Q1 + Q2 = 7.68 C + 2.88 C = 4.80 C 4.8 C

Thus 4.8 C of charge has passed through the switch, from right to left.
59.

(a)

The full-scale current is the reciprocal of the sensitivity.


I full =
scale

(b)

1
= 2.9 105 A or 29 A
35,000 /V

The resistance is the full-scale voltage multiplied by the sensitivity.


R = Vfull (sensitivity) = (250 V)(35,000 /V) = 8.75 106 8.8 106
scale

Copyright 2014 Pearson Education, Inc. All rights reserved. This material is protected under all copyright laws as they currently exist.
No portion of this material may be reproduced, in any form or by any means, without permission in writing from the publisher.

19-28

60.

Chapter 19

The total resistance with the ammeter present is Req = 720 + 480 + 53 = 1253 . The voltage
supplied by the battery is found from Ohms law to be Vbattery = IReq = (5.25 103 A)(1253 )
= 6.578 V. When the ammeter is removed, we assume that the battery voltage does not change. The
= 1200 , and the new current is again found from Ohms law.
equivalent resistance changes to Req
I=

61.

Vbattery

Req

To make a voltmeter, a resistor Rser must be placed


in series with the existing meter so that the desired
full-scale voltage corresponds to the full-scale current
of the galvanometer. We know that 35 mA produces
full-scale deflection of the galvanometer, so the
voltage drop across the total meter must be 25 V
when the current through the meter is 35 mA.
Vfull = I full Req
scale

scale

Vfull
Rser =

scale

I full

scale

(a)

Rser

scale

Rshunt

Vfull

scale

25 V
35 103

1
1

A 33 0.20

= 714.1 710

To make an ammeter, a shunt resistor must be placed in parallel with the galvanometer. The
voltage across the shunt resistor must be the voltage across the galvanometer. See Fig. 1931 for
a circuit diagram.

Rshunt =

(I I G ) Rshunt = I G r

IG r
(55 10 A)(32 )
=
= 7.0 105
(I I G ) (25 A 55 106 A)

To make a voltmeter, a resistor must be placed in series with the galvanometer, so that the desired
full-scale voltage corresponds to the full-scale current of the galvanometer. See Fig. 1932 for a
circuit diagram.
Vfull scale = I G (Rser + r ) Rser =

63.

I full

714
= 29 /V .
25 V

Vshunt = VG

(b)

1
1

= I full Rser + +

r Rshunt
scale

1
1
+

r Rshunt

The sensitivity is
62.

6.578 V
= 5.48 103 A = 5.48 mA
1200

Vfull scale
250 V
r =
32 = 4.5 106
IG
55 106 A

Find the equivalent resistance for the entire circuit, and


then find the current drawn from the source. That current
will be the ammeter reading. The ammeter and voltmeter
symbols in the diagram below are each assumed to have
resistance.

A 0.50
7.5k

7.5k
V
15k

Copyright 2014 Pearson Education, Inc. All rights reserved. This material is protected under all copyright laws as they currently exist.
No portion of this material may be reproduced, in any form or by any means, without permission in writing from the publisher.

DC Circuits

Req = 1.0 + 0.50 + 7500 +

19-29

(7500 )(15, 000 )


(7500 + 15, 000 )

= 12501.5 12500 ; I source =

12.0 V
=
= 9.60 104 A
Req 12500

The voltmeter reading will be the source current times the equivalent resistance of the resistor
voltmeter combination.
Vmeter = I source Req = (9.60 104 A)

(7500 )(15, 000 )


= 4.8 V
(7500 + 15, 000 )

If there were no meters at all, then the equivalent resistance, delivered current, and voltage across the
resistor would be as follows.
Req = 1.0 + 7500 + 7500 = 15,001
I source =

12.0 V

=
= 8.00 104 A
Req 15,001

V7500 = I source R = (8.00 104 A)(7500 ) = 6.00 V

We also calculated the % difference from the readings with realistic meters.
9.60 104 A 8.00 104 A
(% diff) I =
100 = 20 %

8.00 104 A

4.8 V 6.0 V
(% diff)V =
100 = 20 %
6.0 V

The current reads 20% higher with the meters present, and the voltage reads 20% lower.
64.

We know from Example 1917 that the voltage across the resistor without the voltmeter connected is
4.0 V. Thus the minimum acceptable voltmeter reading is 95% of that: (0.95)(4.0 V) = 3.8 V. The
voltage across the other resistor would then be 4.2 V, which is used to find the current in the circuit.
I=

V2
4.2 V
=
= 2.8 1024 A
R2 15, 000

This current is used with the voltmeter reading to find the equivalent resistance of the meterresistor
combination, from which the voltmeter resistance can be found.
V
3.8 V
= 13,570
Rcomb = comb =
I
2.8 104 A
1
Rcomb

Rmeter =

65.

1
1
+
R1 Rmeter

1
Rmeter

1
Rcomb

1
R1

R1 Rcomb
(15,000 )(13,570 )
=
= 142,300 140 k
R1 Rcomb 15,000 13,570

The sensitivity of the voltmeter is 1000 ohms per volt on the 3.0volt scale, so it has a resistance of 3000 ohms. The circuit is shown
in the diagram. Find the equivalent resistance of the meterresistor
parallel combination and the entire circuit.

R
V
RV

Copyright 2014 Pearson Education, Inc. All rights reserved. This material is protected under all copyright laws as they currently exist.
No portion of this material may be reproduced, in any form or by any means, without permission in writing from the publisher.

19-30

Chapter 19
1

1
RV R
1
(3000 )(9400 )
Rp = +
=
= 2274
=
R
R
R
R
3000 + 9400
+
V
V

Req = R + Rp = 2274 + 9400 = 11, 674

Using the meter reading of 1.9 volts, calculate the current into the parallel combination, which is the
current delivered by the battery. Use that current to find the emf of the battery.
I=

1.9 V
V
=
= 8.355 104 A
Rp 2274

= IReq = (8.355 104 A)(11, 674 ) = 9.754 V 9.8 V

66.

Because the voltmeter is called high resistance, we can assume it has no current passing through it.
Write Kirchhoffs loop rule for the circuit for both cases, starting with the negative pole of the battery
and proceeding counterclockwise.
Case 1: Vmeter = V1 = I1 R1

I1r I1 R1 = 0 = I1 (r + R1 ) =

Case 2: Vmeter = V2 = I 2 R2

V1
(r + R1 )
R1

I 2 r I 2 R2 = 0 = I 2 (r + R2 ) =

V2
(r + R2 )
R2

Solve these two equations for the two unknowns of and r.


=

V1
V
(r + R1 ) = 2 (r + R2 )
R1
R2

V2 V1

8.1 V 9.7 V
r = R1 R2
= (35 )(14.0 )
= 5.308 5.3
(9.7 V)(14.0 ) (8.1 V)(35 )
V1 R2 V2 R1
V
9.7 V
= 1 (r + R1 ) =
(5.308 + 35 ) = 11.17 V 11.2 V
R1
35

67.

We can use a voltage divider circuit, as discussed in Example 193b and illustrated in Fig. 196b. The
current to be passing through the body is given by Ohms law.
I=

Vbody
Rbody

0.25 V
= 1.389 104 A
1800

This is the current in the entire circuit. Use this to find the resistor to put in series with the body.
Vbattery = I (Rbody + Rseries )

Rseries =

68.

(a)

Vbattery
I

Rbody =

1.5 V
1.389 104 A

1800 = 8999 = 9.0 k

Since P = V 2 /R and the voltage is the same for each combination, the power and resistance are
inversely related to each other. So for the 50-W output, use the higher-resistance filament. For
the 100-W output, use the lower-resistance filament. For the 150-W output, use the filaments in
parallel.

Copyright 2014 Pearson Education, Inc. All rights reserved. This material is protected under all copyright laws as they currently exist.
No portion of this material may be reproduced, in any form or by any means, without permission in writing from the publisher.

DC Circuits

(b)

P = V 2 /R R =

V2
P

R1 =

(120 V) 2
= 288 290
50 W

R2 =

19-31

(120 V) 2
= 144 140
100 W

As a check, the parallel combination of the resistors gives the following.


Rp =

69.

R1 R2
(288 )(144 )
V 2 (120 V) 2
=
= 96 P =
=
= 150 W
R1 + R2
288 + 144
R
96

There are nine values of effective capacitance that can be obtained from the four capacitors.
All four in parallel:

All four in series:

Ceq = C + C + C + C = 4C

1 1 1 1
Ceq = + + +
C C C C

1C
4

(Three in parallel) in series with one:


1
1
1
1 1
4
=
+ =
+ =
Ceq C + C + C C 3C C 3C

Ceq =

3C
4

(Two in parallel) in series with (two in parallel):


1
1
1
1
1
2
=
+
=
+
=
Ceq C + C C + C 2C 2C 2C

Ceq = C

(Two in parallel) in series with (two in series):


1
1
1 1
1
2
5
=
+ + =
+ =
Ceq C + C C C 2C C 2C

Ceq =

2C
5

(Two in series) in parallel with (two in series):


1 1
Ceq = +
C C

1 1
+ +
C C

2
=
C

2
+
C

C C
+ =C
2 2

(not a new value)


(Three in series) in parallel with one:
1 1 1
Ceq = + +
C C C

3
+C =
C

+C =

4C
3

(Two in series) in parallel with (two in parallel):


1 1
Ceq = +
C C

2
+C +C =
C

+ 2C =

5C
2

Copyright 2014 Pearson Education, Inc. All rights reserved. This material is protected under all copyright laws as they currently exist.
No portion of this material may be reproduced, in any form or by any means, without permission in writing from the publisher.

19-32

Chapter 19

((Two in series) in parallel with one) in series with one:


1

1
1 1 1

1
1 C
1

= + + C + = + C +
Ceq C C
C 2
C

2 1
5
=
+ =
Ceq = 53 C
3C C 3C

((Two in parallel) in series with one) in parallel with one:


1
1
Ceq = +

C 2C

70.

+C = C +

2C
=
3

5C
3

The equivalent resistance is the sum of all the resistances. The current is found from Ohms law.
I=

240 V
=
= 8 103 A
Req 3 104

This is 8 milliamps, and 10 milliamps is considered to be a dangerous level in that it can cause sustained
muscular contraction. The 8 milliamps could certainly be felt by the patient and could be painful.
71.

The capacitor will charge up to 75% of its maximum value and then discharge. The charging time is
the time for one heartbeat.
tbeat =

1 min
60 s

= 0.8333 s
72 beats 1 min

beat
0.75 V0 = V0 1 e RC

tbeat
0.8333 s
=
=
R=
C ln(0.25)
(6.5 106 F)( 1.3863)

V = V0 1 e

72.

(a)

t
RC

t
t

beat
RC = 0.25 beat

RC

= ln(0.25)

9.2 104

Apply Ohms law to find the current.


I=

Vbody
Rbody

120 V
= 0.126 A 0.13 A
950

(b)

The description of alternative path to ground is a statement that the 25- path is in parallel
with the body. Thus the full 110 V is still applied across the body, so the current is the same:
0.13 A .

(c)

If the current is limited to a total of 1.5 A, then that current will be divided between the person
and the parallel path. The voltage across the body and the parallel path will be the same, since
they are in parallel.
Vbody = Valternate
I body = I total

I body Rbody = I alternate Ralternate = (I total I body )Ralternate

Ralternate
25
= (1.5 A)
= 0.03846 A 38 mA
(Rbody + Ralternate )
950 + 25

This is still a very dangerous current.

Copyright 2014 Pearson Education, Inc. All rights reserved. This material is protected under all copyright laws as they currently exist.
No portion of this material may be reproduced, in any form or by any means, without permission in writing from the publisher.

DC Circuits

73.

The original resistance of the motor is found from Ohms law.


V = IR Roriginal =

(a)

V 12 V
=
= 2.4
I 5.0 A

Now another resistor is put in series to reduce the current to 2.0 A. We assume the battery
voltage has not changed.
V = IR Roriginal + Rseries =

74.

V 12 V
=
= 6.0 Rseries = 6.0 2.4 = 3.6
I 2.0 A

(b)

P = I 2 R = (2.0 A) 2 (3.6 ) = 14.4 W 14 W

(a)

If the ammeter shows no current with the closing of the switch, then points B and D must be at
the same potential, because the ammeter has some small resistance. Any potential difference
between points B and D would cause current to flow through the ammeter. Thus the potential
drop from A to B must be the same as the drop from A to D. Since points B and D are at the
same potential, the potential drop from B to C must be the same as the drop from D to C. Use
these two potential relationships to find the unknown resistance.

(b)
75.

19-33

VBA = VDA

I 3 R3 = I1 R1

VCB = VCD

I 3 Rx = I1 R2

Rx = R2

R3 I1
=
R1 I 3

Rx = R2

I1
= R2 R3 /R1
I3

78.6
R3
= (972 )
= 129 130
R1
590

Divide the power by the required voltage to determine the current drawn by the hearing aid.
I=

P 2.5 W
=
= 0.625 A
V 4.0 V

Use Eq. 191 to calculate the terminal voltage across the series combinations of three batteries for both
mercury (Hg) and dry (D) cells.
VHg = 3( Ir ) = 3[1.35 V (0.625 A)(0.030 )] = 3.99 V
VD = 3( Ir ) = 3[1.50 V (0.625 A)(0.35 )] = 3.84 V

The terminal voltage of the mercury cell batteries is closer to the required 4.0 V than the voltage from
the dry cell.
76.

One way is to connect N resistors in series. If each resistor can dissipate 0.5 W, then it will take 7
resistors in series to dissipate 3.5 W. Since the resistors are in series, each resistor will be 1/7 of the
total resistance.
R=

Req
7

3200
= 457 460
7

So connect 7 resistors of 460 each, rated at W, in series. Each resistor would have to have a
voltage of about 15 V in order to not exceed the power rating. Thus the total voltage would be limited
to about 105 V.
Copyright 2014 Pearson Education, Inc. All rights reserved. This material is protected under all copyright laws as they currently exist.
No portion of this material may be reproduced, in any form or by any means, without permission in writing from the publisher.

19-34

Chapter 19

Alternately, the resistors could be connected in parallel. Again, if each resistor watt can dissipate
0.5 W, then it will take 7 resistors in parallel to dissipate 3.5 W. Since the resistors are in parallel, the
equivalent resistance will be 1/7 of each individual resistance.
1
1
= 7 R = 7 Req = 7(3200 ) = 22.4 k
Req
R

So connect 7 resistors of 22.4 k each, rated at W, in parallel. Each resistor could have about
105 V across it in this configuration.
77.

To build up a high voltage, the cells will have to be put in series. 120 V is needed from a series of
120 V
0.80 V cells. Thus
= 150 cells are needed to provide the desired voltage. Since these cells
0.80 V/cell
are all in series, their current will all be the same at 350 mA. To achieve the higher current desired,
banks made of 150 cells each can be connected in parallel. Then their voltage will still be at 120 V, but
1.3 A
the currents would add, making a total of
= 3.71 banks 4 banks. So the total
350 103 A/bank
number of cells is 600 cells . The panel area is 600 cells (9.0 104 m 2 /cell) = 0.54 m 2 . The cells
should be wired in 4 banks of 150 cells in series per bank, with the banks in parallel. This will produce
1.4 A at 120 V.

78.

There are two answers because it is not known which direction the given current is flowing through the
4.0-k resistor. Assume the current is to the right. The voltage across the 4.0-k resistor is given by
Ohms law as V = IR = (3.10 103 A)(4000 ) = 12.4 V. The voltage drop across the 8.0 k must be
the same, so the current through it is I =

V 12.4 V
=
= 1.55 103 A. The total current in the circuit is
R 8000

the sum of the two currents: I tot = 4.65 103 A. That current can be used to find the terminal voltage
of the battery. Write a loop equation, starting at the negative terminal of the unknown battery and
going clockwise.
Vba (3200 )I tot 12.4 V 12.0 V (1.0 )I tot
Vba = 24.4V + (3201 )(4.65 10

A) = 39.28 V 39 V

If the current is to the left, then the voltage drop across the parallel combination of resistors is still
12.4 V, but with the opposite orientation. Again write a loop equation, starting at the negative terminal
of the unknown battery and going clockwise. The current is now to the left.
Vab + (3200 )I tot + 12.4 V 12.0 V + (1.0 )I tot

Vab = 0.4V (3201 )(4.65 103 A) = 15.28 V 15 V

79.

(a)

If the terminal voltage is to be 3.5 V, then the voltage across R1 will be 8.5 V. This can be used
to find the current, which then can be used to find the value of R2 .
V
V1 = IR1 I = 1 V2 = IR2
R1
R2 =

V2
V
3.5 V
= R1 2 = (14.5 )
= 5.971 6.0
I
V1
8.5 V

Copyright 2014 Pearson Education, Inc. All rights reserved. This material is protected under all copyright laws as they currently exist.
No portion of this material may be reproduced, in any form or by any means, without permission in writing from the publisher.

DC Circuits

(b)

19-35

If the load has a resistance of 7.0 , then the parallel combination of R2 and the load must be
used to analyze the circuit. The equivalent resistance of the circuit can be found and used to
calculate the current in the circuit. Then the terminal voltage can be found from Ohms law,
using the parallel combination resistance.
R2+ load =
I=

R2 Rload
(5.971 )(7.0 )
=
= 3.222 Req = 3.222 + 14.5 = 17.722
R2 + Rload (5.971 + 7.0 )

V
12.0 V
=
= 0.6771 A VT = IR2+ load = (0.6771 A)(3.222 ) = 2.182 V 2.2 V
Req 17.722

The presence of the load has affected the terminal voltage significantly.
80.

The terminal voltage and current are given for two situations. Apply Eq. 191 to both of these
situations and solve the resulting two equations for the two unknowns.
V1 = I1r ; V2 = I 2 r = V1 + I1r = V2 + I 2 r
r=

V2 V1 47.3 V 40.8 V
=
= 1.161 1.2
I1 I 2 8.40 A 2.80 A

= V1 + I1r = 40.8 V + (8.40 A)(1.161 ) = 50.6 V

81.

The current in the circuit can be found from the resistance and the power dissipated. Then the product
of that current and the equivalent resistance is equal to the battery voltage.
P = I 2R I =

P33
0.80 W
=
= 0.1557 A
33
R33

1
1
+
Req = 33 +

68
85

82.

= 70.78 V = IReq = (0.1557 A)(70.78 ) = 11.02 V 11 V

There are three loops, so there are three loop


equations necessary to solve the circuit. We
identify five currents in the diagram, so we need
two junction equations to complete the analysis.
We also assume that the 10-k resistor is known
to 2 significant digits, like the other resistors.
Finally, note that if we use the resistances in kiloohms, the currents will be in milli-amps.
Lower left junction:

I1

I4

I5

I2

I3

I 4 = I1 + I 3

Lower right junction: I 2 = I3 + I 5


Left loop, clockwise:

16 V 10I1 12I 4 = 0

Right loop, counterclockwise:

21 V 13I 2 18I 5 = 0

Bottom loop, counterclockwise: 12 V + 18I 5 12I 4 = 0

Copyright 2014 Pearson Education, Inc. All rights reserved. This material is protected under all copyright laws as they currently exist.
No portion of this material may be reproduced, in any form or by any means, without permission in writing from the publisher.

19-36

Chapter 19

16 12I 3
22
21 13I 3
21 13(I 3 + I 5 ) 18I5 = 0 21 13I 3 31I5 = 0 I 5 =
31
12 + 18I 5 12(I1 + I 3 ) = 0 12 12 I1 12 I 3 + 18 I5 = 0

16 10I1 12(I1 + I 3 ) = 0 16 22I1 12I 3 = 0 I1 =

Substitute the results of the first two equations into the third equation.
16 12 I 3
21 13I 3
12 12
12 I 3 + 18 31 = 0
22

12 12
(16 12 I 3 ) 12 I 3 + 18
(21 13I 3 ) = 0
22
31
6 16 + 18 21 = (12 + 18 13 6 12)I
12 11
3
31
31
11

15.466 = 13.003I 3

I3 =

15.466
= 1.189 mA
13.003

16 12I 3 16 12(1.189)
=
= 0.0787 mA 7.9 105 A, upward
22
22

(a)

I1 =

(b)

Reference all voltages to the lower left corner of the circuit diagram. Then Va = 16 V,
Vb = 12 V + 21 V = 33 V, and Va Vb = 16 V 33 V = 17 V .

83.

If the switches are both open, then the circuit is a simple series circuit. Use Kirchhoffs loop rule to
find the current in that case.
6.0 V I (50 + 20 + 10 ) = 0 I = 6.0 V/80 = 0.075 A

If the switches are both closed, then the 20- resistor is in parallel with R. Apply Kirchhoffs loop
rule to the outer loop of the circuit, with the 20- resistor having the current found previously.
6.0 V I (50 ) (0.075 A)(20 ) = 0 I =

6.0 V (0.075 A)(20 )


= 0.090 A
50

This is the current in the parallel combination. Since 0.075 A is in the 20- resistor, 0.015 A must be
in R. The voltage drops across R and the 20- resistor are the same since they are in parallel.
V20 = VR

84.

(a)

I 20 R20 = I R R R = R20

I 20
0.075 A
= (20 )
= 100
IR
0.015 A

The 12- and the 22- resistors are in parallel, with a net resistance R1 2 as follows.
1
1
R1 2 =
+

12 22

= 7.765

R1 2 is in series with the 4.5- resistor, for a net resistance R1 23 as follows.


R1 23 = 4.5 + 7.765 = 12.265

Copyright 2014 Pearson Education, Inc. All rights reserved. This material is protected under all copyright laws as they currently exist.
No portion of this material may be reproduced, in any form or by any means, without permission in writing from the publisher.

DC Circuits

19-37

That net resistance is in parallel with the 14- resistor, for a final equivalent resistance as
follows.

1
1
Req =
+

12.265
14

(b)

Find the current in the 14- resistor by using Kirchhoffs loop rule for the loop containing the
battery and the 14- resistor.
I14 R14 = 0 I18 =

(c)

= 6.538 6.5

6.0 V
=
= 0.4286 A 0.43 A
R18 14

Find the current in R1 2 and the 4.5- resistor by using Kirchhoffs loop rule for the outer loop
containing the battery and the resistors R1 2 and the 4.5- resistor. Those resistances are in
series.
I1 2 R1 2 I1 2 R4.5 = 0 I1 2 =

6.0 V
=
= 0.4892 A
R1 2 + R4.5 7.765 + 4.5

This current divides to go through the 12- and 22- resistors in such a way that the voltage
drop across each of them is the same. Use that to find the current in the 12- resistor.
I1 2 = I12 + I 25
VR12 = VR25
I12 = I1 2

(d)

I 25 = I1 2 I12

I12 R12 = I 25 R25 = (I1 2 I12 )R25

R25
22
= (0.4892 A)
= 0.3165 A 0.32 A
(R12 + R25 )
34

The current in the 4.5- resistor was found above to be I1 2 = 0.4892 A. Find the power
accordingly.
P4.5 = I12 2 R4.5 = (0.4892 A) 2 (4.5 ) = 1.077 W 1.1 W

85.

(a)

We assume that the ammeter is ideal so has 0 resistance but that the voltmeter has resistance
RV . Then apply Ohms law, using the equivalent resistance. We also assume the voltmeter is
accurate, so it is reading the voltage across the battery.
V = IReq = I

(b)

1
1
1
+
R RV

1
1
V +
=I
R RV

1
1
I
+
=
R RV V

1 I
1
=
R V RV

We now assume the voltmeter is ideal, so has an infinite resistance but that the ammeter has
resistance RA . We also assume that the voltmeter is accurate and is reading the voltage across
the battery.
V = IReq = I (R + RA ) R + RA =

V
I

R=

V
RA
I

Copyright 2014 Pearson Education, Inc. All rights reserved. This material is protected under all copyright laws as they currently exist.
No portion of this material may be reproduced, in any form or by any means, without permission in writing from the publisher.

19-38

86.

Chapter 19

(a)

The light will first flash when the voltage across the capacitor reaches 90.0 V.
t

RC


V = 0 1 e

90
V

6
6
t = RC ln 1
= (2.35 10 )(0.150 10 F) ln 1
= 0.686 s
105
0

(b)

We see from the equation that t R, so if R increases, then the time will increase.

(c)

The capacitor discharges through a very low resistance (the lamp filled with ionized gas), so the
discharge time constant is very short. Thus the flash is very brief.
Once the lamp has flashed, the stored energy in the capacitor is gone, and there is no source of
charge to maintain the lamp current. The lamp goes out, the lamp resistance increases, and the
capacitor starts to recharge. It charges again for 0.686 second, and the process will repeat.

(d)

87.

We find the current for the bulb from the bulbs ratings. Also, since the bulb will have 3.0 V across it,
the resistor R will have 6.0 V across it.
Pbulb = I bulbVbulb

88.

(a)

I bulb =

Pbulb 2.0 W
V
6.0 V
=
= 0.667 A; R = R =
= 9.0
Vbulb 3.0 V
I R 0.667 A

As shown, the equivalent capacitance is found from adding the capacitance of C1 to the series
combination of C1 and C1. That net capacitance is used to calculate the stored energy.
1
1
Cnet = C1 +
+

C
C
3
2

2
= 25.4 F +

25.4
F

= 38.1 F

PE = 12 CnetV 2 = 12 (38.1 106 F)(10.0 V) 2 = 1.905 103 J 1.91 103 J

(b)

If all the capacitors were in series, then the equivalent capacitance is 1/3 the value of one of the
capacitors.
1
1
1
Cnet = +
+

C
C
C
2
3
1

3
=

25.4
F

= 8.467 F

PE = 12 CnetV 2 = 12 (8.467 106 F)(10.0 V) 2 4.24 104 J

(c)

If all the capacitors were in parallel, then the equivalent capacitance is 3 times the value of one
of the capacitors.
Cnet = C1 + C2 + C3 = 3(25.4 F) = 76.2 F
PE = 12 CnetV 2 = 12 (76.2 106 F)(10.0 V) 2 3.81 104 J

89.

After a long time, the only current is in the resistors. The voltage across the 3.3-k resistor is found
using the left loop.
12.0V I (4600 ) = 0 I =

12.0V
= 2.609 103
4600

V3.3 k = IR = (2.609 103 )(3300 ) = 8.609 V

Copyright 2014 Pearson Education, Inc. All rights reserved. This material is protected under all copyright laws as they currently exist.
No portion of this material may be reproduced, in any form or by any means, without permission in writing from the publisher.

DC Circuits

19-39

The voltage across the 3.3 k is the voltage across each capacitor.
Q12 F = CV = (12 106 F)(8.609 V) = 1.033 104 C 1.0 104 C
Q48 F = CV = (48 106 F)(8.609 V) = 4.132 104 C 4.1 104 C

90.

We have labeled currents through the resistors


with the value of the specific resistance and the
emfs with the appropriate voltage value. We
apply the junction rule to points a and b and
then apply the loop rule to the left loop, the middle
loop, and the right loop. This enables us to solve
for all of the currents.
I 5 = I 6 + I top ; I top + I 6.8 = I12

I 5 I 6 = I12 I 6.8

17 V I 5 (5.00 ) I 6 (6.00 ) = 0

I6

I5

I top

I12

I 6.8

I 5 + I 6.8 = I12 + I 6 [1]

[2] (left loop)

10 V I12 (12.00 ) I 6.8 (6.80 ) = 0 [3] (right loop)

I12 (12.00 ) I 6 (6.00 ) = 0

[4] (center loop)

Solve Eq. [4] to get I 6 = 2 I12 . Substitute that into the other equations.
I 5 + I 6.8 = 3I12

[1]

17 V I5 (5.00 ) 2 I12 (6.00 ) = 0


10 V I12 (12.00 ) I 6.8 (6.80 ) = 0

[ 2]
[3]

Use Eq. [1] to eliminate I 6.8 by I 6.8 = 3I12 I 5 .


17 V I5 (5.00 ) I12 (12.00 ) = 0

[ 2]

10 V I12 (12.00 ) (3I12 I 5 )(6.80 ) = 0 10 V I12 (32.40 ) + I 5 (6.80 ) = 0

Use Eq. [2] to eliminate I 5 by I 5 =

[3]

17 V I12 (12.00 )
, and then substitute into Eq. [3] and solve for
(5.00 )

solve for I12 . Then back-substitute to find the other currents.


17 V I12 (12.00 )
(6.80 ) = 0
(5.00 )

10 V I12 (32.40 ) +

10 V(5.00 ) I12 (32.40 )(5.00 ) + [17 V I12 (12.00 ) ] (6.80 ) = 0


I12 =

I5 =

10 V(5.00 ) + 17 V(6.80 )

(12.00 )(6.80 ) + (32.40 )(5.00 )

= 0.6798 A

I12 = 0.680 A

17 V I12 (12.00 ) 17 V (0.6798 A)(12.00 )


=
= 1.768 A I 5 = 1.77 A
(5.00 )
(5.00 )

I 6.8 = 3I12 I 5 = 3(0.6798 A) 1.768 A = 0.2714 A I 6.8 = 0.271 A


I 6 = 2 I12 = 2(0.6798 A) = 1.3596 A I 6 = 1.36 A

Copyright 2014 Pearson Education, Inc. All rights reserved. This material is protected under all copyright laws as they currently exist.
No portion of this material may be reproduced, in any form or by any means, without permission in writing from the publisher.

19-40

91.

Chapter 19

(a)

When the capacitors are connected in parallel, their equivalent capacitance is found from Eq. 195.
The stored energy is given by Eq. 1710.
Ceq = C1 + C2 = 0.65 F; PE = 12 CeqV 2 = 12 (0.65 106 F)(24 V) 2 = 1.9 104 J

(b)

When the capacitors are connected in parallel, their equivalent capacitance is found from Eq. 196.
The stored energy is again given by Eq. 1710.
1
1
1
=
+
Ceq C1 C2

Ceq =

C1C2
(0.45 F)(0.20 F)
=
= 0.1385 F
C1 + C2
0.65 F

PE = 12 CeqV 2 = 12 (0.1385 106 F)(24 V) 2 = 4.0 105 J

The charge is found from Eq. 177, Q = CV .

(c)

Qparallel = (Ceq ) parallelV = (0.65 F)(24 V) = 16 C


Qseries = (Ceq )seriesV = (0.1385 F)(45 V) = 3.3 C

92.

Because Q = CV and the capacitors both have the same voltage applied, it will be the case that
Q1 > Q2 . Because of conservation of charge, when the capacitors are hooked up, the total charge that
is available is Q1 Q2 . That amount of charge redistributes over the capacitor plates, and once
equilibrium has been reached, the total charge on the plates is given by q1 + q2 = Q1 Q2 . The other
constraint on this system is that once equilibrium has been reached, the capacitors must have the same
q
q
voltage, so 1 = 2 .
C1 C2

Q1 = C1V = (2.2 F)(24 V) = 52.8 F; Q2 = C2V = (1.2 F)(24 V) = 28.8 F


Q1 Q2 = 52.8 F 28.8 F = 24.0 F = q1 + q2
q1
C1

q2
C2

24.0 F q1

q1 = 15.5 C,

93.

C2

q1 =

24.0 F

C2
1 +
C1

q2 = 24.0 F q1

24.0 F

1.2 F
1 + 2.2 F

= 15.5 C

q2 = 8.5 C

When the switch is down, the initial charge on C2 is calculated from Eq. 177.
Q2 = C2V0

When the switch is moved up, charge will flow from C2 to C1 until the voltage
across the two capacitors is equal.
V=

Q2 Q1
=
C2 C1

Q2 = Q1

C2
C1

Copyright 2014 Pearson Education, Inc. All rights reserved. This material is protected under all copyright laws as they currently exist.
No portion of this material may be reproduced, in any form or by any means, without permission in writing from the publisher.

DC Circuits

19-41

The sum of the charges on the two capacitors is equal to the initial charge on C2 .
Q2 = Q2 + Q1 = Q1

C + C2
C2
+ Q1 = Q1 1

C1
C1

Inserting the initial charge in terms of the initial voltage gives the final charges.
C + C2
Q1 1
= C2V0
C1

94.

Q1 =

C1C2
C
C22
V0 ; Q2 = Q1 2 =
V0
C1 + C2
C1
C1 + C2

The power delivered to the starter is equal to the square of the current in the circuit multiplied by the
resistance of the starter. Since the resistors in each circuit are in series, we calculate the currents as the
battery emf divided by the sum of the resistances.
2
/Req
P I 2 RS I
= 2 = =
/R0eq
P0 I 0 RS I 0

R0eq
=

Req

r + RS
=

r + RS + RC

0.02 + 0.15
=
= 0.40
0.02
0.15
0.10

95.

(a)

From the diagram, we see that one group of four plates is connected together, and the other
group of four plates is connected together. This common grouping shows that the capacitors are
connected in parallel. Each capacitor would have the same voltage across it.

(b)

Since they are connected in parallel, the equivalent capacitance is the sum of the individual
capacitances. The variable area will change the equivalent capacitance.
Ceq = 7C = 7 0

A
d

Cmin = 7 0

Amin
(2.0 104 m 2 )
= 7(8.85 1012 C2 /N m 2 )
= 7.7 1012 F
3
d
(1.6 10 m)

Cmax = 7 0

Amax
(9.0 104 m 2 )
= 7(8.85 1012 C2 /N m 2 )
= 3.5 1011 F
3

d
(1.6 10 m)

So the range is from 7.7 pF to 35 pF .


96.

Capacitors in series each store the same amount of charge, so the capacitance on the unknown
capacitor is also 125 pC. We calculate the voltage across each capacitor, and then calculate the
unknown capacitance.
V175 =

Q175 125 1012 C


=
= 0.714 V
C175 175 1012 F

Vunknown = 25.0 V V185 = 25.0 V 0.714 V = 24.286 V


Cunknown =

Q 125 1012 C
=
= 5.15 1012 C
V
24.286 V

Copyright 2014 Pearson Education, Inc. All rights reserved. This material is protected under all copyright laws as they currently exist.
No portion of this material may be reproduced, in any form or by any means, without permission in writing from the publisher.

19-42

Chapter 19

Thus the voltage across the unknown capacitor is 25 V V185 = 25 V 0.676 V = 24.324 V. The
capacitance can be calculated from the voltage across and charge on that capacitor.
C=

97.

Q 125 1012 C
=
= 5.14 pF
V
24.324 V

The first capacitor is charged and has charge Q0 = C1V0 on its plates. Then, when the switch is moved,
the capacitors are not connected to a source of charge, so the final charge is equal to the initial charge.
Initially treat capacitors C2 and C3 as their equivalent capacitance,
C23 =

C2C3
(2.0 F)(2.4 F)
=
= 1.091 F. The final voltage across C1 and C23 must be the same.
C2 + C3
4.4 F

The charge on C2 and C3 must be the same. Use Eq. 177.


Q0 = C1V0 = Q1 + Q23 = C1V1 + C23V23 = C1V1 + C23V1
V1 =

C1
1.0 F
(24 V) = 11.48 V = V1 = V23
V0 =
1.0 F + 1.091 F
C1 + C23

Q1 = C1V1 = (1.0 F)(11.48 V) = 11.48 C


Q23 = C23V23 = (1.091 F)(11.48 V) = 12.52 C = Q2 = Q3
V2 =

Q 12.52 C
Q2 12.52 C
=
= 6.26 V; V3 = 3 =
= 5.22 V
2.0 F
2.4 F
C2
C3

To summarize: Q1 = 11 C, V1 = 11 V; Q2 = 13 C, V2 = 6.3 V; Q3 = 13 C, V3 = 5.2 V .

Solutions to Search and Learn Problems


1.

Req

Ceq

Series

Req = R1 + R2 + "

1
1
1
=
+
+"
Ceq C1 C2

Parallel

1
1
1
=
+
+"
Req R1 R2

Ceq = C1 + C2 + "

When resistors are connected in series, the equivalent resistance is the sum of the individual
resistances, Req = R1 + R2 + " . The current has to go through each additional resistance if the
series

resistors are in series and therefore the equivalent resistance is greater than any individual resistance.
In contrast, when capacitors are in parallel, the equivalent capacitance is equal to the sum of the
= C1 + C2 + ". Charge drawn from the battery can go down any one of
individual capacitors, Ceq
parallel

the different branches and land on any one of the capacitors, so the overall capacitance is greater than
that of each individual capacitor.
When resistors are connected in parallel, the current from the battery or other source divides into the
different branches so the equivalent resistance is less than any individual resistor in the circuit. The
Copyright 2014 Pearson Education, Inc. All rights reserved. This material is protected under all copyright laws as they currently exist.
No portion of this material may be reproduced, in any form or by any means, without permission in writing from the publisher.

DC Circuits

1
Req

corresponding expression is

parallel

19-43

1
1
+
+ ". The formula for the equivalent capacitance of
R1 R2

capacitors in series follows this same form,

1
Ceq

series

1
1
+
+ ". When capacitors are in series, the
C1 C2

overall capacitance is less than the capacitance of any individual capacitor. Charge leaving the first
capacitor lands on the second rather than going straight to the battery, so on.
Compare the expressions defining resistance (R = V /I ) and capacitance (C = Q /V ). Resistance is
proportional to voltage, whereas capacitance is inversely proportional to voltage.
2.

Property
Equivalent resistance

Resistors in Series
R1 + R2

Resistors in Parallel
R1 R2 /(R1 + R2 )

Current through
equivalent resistance
Voltage across equivalent
resistance
Voltage across the pair of
resistors
Voltage across each
resistor

(VB VA )/(R1 + R2 )

(VB VA )(R1 + R2 )/(R1 R2 )

VB VA

VB VA

VB VA

VB VA

V1 = (VB VA )R1 /(R1 + R2 )

V1 = VB VA

V2 = (VB VA )R2 /(R1 + R2 )

V2 = VB VA

(VA R2 + VB R1 )/(R1 + R2 )

Not applicable

I1 = (VB VA )/(R1 + R2 )

I1 = (VB VA )/R1

I 2 = (VB VA )/(R1 + R2 )

I 2 = (VB VA )/R2

Voltage at a point
between the resistors
Current through each
resistor
3.

(a)

Use Eq. 197a to solve for the unknown resistance, where the ratio VC / = 0.95.
0.95 = 1 et /RC

(b)

t
2.0 s
=
= 6.7 105

C ln (1 0.95)
(1.0 10 6 F) ln (0.05)

The defibrillator paddles are usually moistened so that the resistance of the skin is about 1000 .
Use Eq. 197c to solve for the capacitance.

= RC C =
4.

R =2

10 103 s
= 10 106 F
1000

When the galvanometer gives a null reading, no current is passing through the galvanometer or the emf
that is being measured. All of the current is flowing through the slide wire resistance. Application of
the loop rule to the lower loop gives IR = 0, since there is no current through the emf to cause a
voltage drop across any internal resistance in the emf or any resistance in the galvanometer. The

Copyright 2014 Pearson Education, Inc. All rights reserved. This material is protected under all copyright laws as they currently exist.
No portion of this material may be reproduced, in any form or by any means, without permission in writing from the publisher.

19-44

Chapter 19

current flowing through the slide wire resistor is independent of the emf in the lower loop because
no current is flowing through the lower loop. We solve the loop rule equation for the current and set
the currents equal. The resulting equation is solved for the unknown emf.
x IRx = 0; s IRs = 0 I =

5.

x s
=
Rx Rs

R
x = x
Rs

Note that, based on the significant figures of the resistors, that the 1.0- resistor will not change the
equivalent resistance of the circuit as determined by the resistors in the switch bank.
Case 1: n = 0 switch closed. The effective resistance of the circuit is 16.0 k. The current in the
circuit is I =

16 V
= 1.0 mA. The voltage across the 1.0- resistor is V = IR
16.0 k

= (1.0 mA)(1.0 ) = 1.0 mV .

Case 2: n = 1 switch closed. The effective resistance of the circuit is 8.0 k. The current in the
circuit is I =

16 V
= 2.0 mA. The voltage across the 1.0- resistor is V = IR
8.0 k

= (2.0 mA)(1.0 ) = 2.0 mV .

Case 3: n = 2 switch closed. The effective resistance of the circuit is 4.0 k. The current in the
circuit is I =

16 V
= 4.0 mA. The voltage across the 1.0- resistor is V = IR
4.0 k

= (4.0 mA)(1.0 ) = 4.0 mV .


Case 4: n = 3 and n = 0 switches closed. The effective resistance of the circuit is found by the parallel
combination of the 2.0-k and 16.0-k resistors.
1
1
+
Req =

2.0 k 16.0 k

The current in the circuit is I =

= 1.778 k
16 V
= 8.999 mA 9.0 mA. The voltage across the
1.778 k

1.0- resistor is V = IR = (9.0 mA)(1.0 ) = 9.0 mV .

So in each case, the voltage across the 1.0- resistor, if taken in mV, is the expected analog value
corresponding to the digital number set by the switches.

Copyright 2014 Pearson Education, Inc. All rights reserved. This material is protected under all copyright laws as they currently exist.
No portion of this material may be reproduced, in any form or by any means, without permission in writing from the publisher.

S-ar putea să vă placă și